Download as docx, pdf, or txt
Download as docx, pdf, or txt
You are on page 1of 62

Mock1 explained by Dr.

Khalid Saifullah

Mock-june 2013. 1. A 56year old nulliparous woman with a BMI of 37 has a four--‐month
history of blood stained discharge. She is otherwise normal. What is the single most likely
diagnosis?
a. Atrophic Vaginitis
b. Bacterial Vaginosis
c. Endometrial Carcinoma
d. Candidiasis
e. Chlamydia

Ans. The key is C. Endometrial Carcinoma. [Post menopausal blood stained discharge is highly
suggestive of endometrial carcinoma]. [a. Atrophic vaginitis- causes post coital bleeding, not
blood stained discharge. B. Bacterial vaginosis- fishy odour (particularly postcoital odour) and
increased vaginal discharge but bleeding is not a feature. d. Candidiasis- it causes vaginal
erythema and curd-like or cheesy discharge, itching and dyspareunia, but bleeding is not a
feature. e. Chlamydia- postcoital usually when there is ectropion. otherwise no bleeding].

Mock-june 2013. 2. A 23year old woman has been taking the combined oral contraceptive pill
for 4 years. She has a troublesome clear, sticky discharge with no odour or itching. What is the
single most likely diagnosis?
a. Cervical cancer
b. HIV Infection
c. Ectropion
d. Gonorrhea
e. Chlamydia

Ans. The key is C. Ectropion. [Oral contraceptive is a well known cause of cervical ectropion.
There may be increase in vaginal discharge which is otherwise asymptomatic (no redness or
plaque, no itching, no odour or other symptoms) in cervical ectropion].

Mock-june 2013. 3. 3. A 25year old pregnant woman has a slight increase in vaginal discharge
with vulval itching. Her partner has recent balanitis. What is the single most likely diagnosis?
a. Ectropion
b. Cervical polyp
c. Cervical carcinoma
d. Candidiasis
e. Atrophic Vaginitis

Ans. The key is D. Candidiasis. [In candidal vulvovaginitis there may be increased vaginal
discharge which can be thin and watery or thick and white, like cottage cheese. There is vulval
itching. Though it is not regarded as STD but is more common in sexually active patients and
there may be candidal balanitis in male partner].

Mock-june 2013. 4. A 76year old woman has vaginal soreness and an intermittent pink staining
on her underwear. She is not on any medication. What is the single most likely diagnosis?
a. Atrophic vaginitis
b. Bacterial vaginistis
c. Endometrial carcinoma
d. Candidiasis
e. Chlamydia

Ans. The key is A. Atrophic vaginitis. [Vaginal soreness may cause little discharge which may
appear as pink staining on underwear].

Mock-june 2013. 5. A 19--‐year--‐old woman in a new sexual relationship has a vaginal


discharge of sudden onset. There is intense itching at the introitus and examination reveals a
frothy discharge with redness of her vulva. What is the single most likely diagnosis?
a. Syphilis
b. HIV Infection
c. HPV
d. Trichomonas Infection
e. Gonorrhea

Ans. The key is D. Trichomonas Infection. [Vaginal discharge with intense itching, redness of
vulva and frothy discharge occur in trichomoniasis].

Mock-june 2013. 6. A 62year old woman has right-‐sided headaches with malaise, weight loss
and depression. What is the single most appropriate investigation?
  a. Temporal artery biopsy
  b. ESR
  c. Skull X-‐Ray
  d. Nerve conductionstudies
  e. MRI imaging of brain.

Ans. The key is A. Temporal artery biopsy. [Right-sided (one sided) headache with malaise,
weight loss and depression in 62 yrs elderly is highly suggestive of temoral arteritis for which
“SINGLE” most appropriate investigation is temporal artery biopsy. If asked most appropriate
answer would be ESR].

Mock-june 2013. 7. A 56-year-old man has sudden severe occipital headaches, drowsiness and
vomiting. What is the single most initial appropriate investigation?
a. CT scan of brain
b. No investigation required
c. Carotid arteriography
d. Lumber puncture
e. Temporal artery biopsy
Ans. The key is A. CT scan of brain. [Sudden severe occipital headaches, drowsiness and
vomiting are highly suggestive of SAH. Here single most initial appropriate investigation is CT
scan of brain].

Mock-june 2013. 8. A 45-year-old woman has right sided headaches, nausea and visual
disturbance. This lasted for 24 hours and she is fine. She was recently started on hormone
replacement therapy. What is the single most initial appropriate investigation?
a. Temporal artery biopsy
b. ESR
c. No investigation
d. Nerve conduction studies
e. MRI imaging of the brain 

Ans. The key is C. No investigation. [One sided headache lasting for 24 hours associated with
nausea and visual disturbance without any focal neurological sign is highly suggestive of
migraine. So for this “no investigation is needed”].

Mock-june 2013. 9. A 46-year-old man has a headache and confusion three days after slipping
in the garden. What is the single most appropriate investigation?

a. CT scan of the brain


b. No investigation required
c. Carotid arteriogram
d. Lumber puncture
e. Temporal artery biopsy

Ans. The key is A. CT scan of the brain. [Probable diagnosis is subdural hematoma. Slipping in
the garden caused some injury to head which led to subdural hematoma supported by clinical
features of headache and confusion after 3 days. So the investigation of choice in such case is
CT scan of the brain].

Mock-june 2013. 10. A 58-year-old man presents with chest pain, which is aggravated by
exercise but relieved with GTN spray. He has recurrent episodes of pain. Examination is normal
and ECG shows new T wave depression in lead II, III and aVF. What is the single most initial
combination to administer?

a. Antiplatelet drugs and low molecular weight heparin


b. Anti-platelet drugs and pravastin
c. Anti-platelet drugs and streptokinase
d. Low molecular weight heparin and warfarin
e. Low molecular weight heparin and pravastin
Ans. The key is A. Antiplatelet drugs and low molecular weight heparin. [This is a case of
NSTEMI. So the single most initial combination to administer is a. Antiplatelet drugs and low
molecular weight heparin. Low molecular weight heparin will prevent further events and
this important advantage can not be given by any other option].

Mock-june 2013. 11. A 60-year-old woman has increasing frequency and urgency leading to
occasional incontinence for 3 months. Abdominal and renal examination is normal. What is
the most appropriate initial investigation?

a. Flexible cystoscopy
b. Retrograde cystourethrography
c. Ultrasound scan of the kidneys
d. Urinalysis for nitrates and leukocytes
e. Urodynamic studies

Ans. The key is E. Urodynamic studies. [Use of urodynamic studies: To know the causes of-
incontinence, frequency, urgency, problems starting a urine stream, problem emptying
bladder completely, UTI etc.].

Mock-june 2013. 12. A 30 year-old-man has developed a red raised lesion on his trunk and
limbs after playing football one hour ago. He is feeling unwell with a slight cough. He had
asthma, as a child but is otherwise well. The rash is becoming increasingly itchy. What is the
most initial appropriate treatment?

a. Intravenous adrenaline
b. Intravenous dexamethasone
c. Oral prednisolone
d. Oral chlorpheniramine
e. Intravenous penicillin

Ans. The key is D. Oral chlorpheniramine. [The patient developed generalized urticaria and
he has no feature of anaphylaxis. This is treated with oral chlorpheiramine].

Mock-june 2013. 13. A 38-year-old woman has low mood, early morning awakening and
poor appetite for six weeks. She had a similar episode 12 years ago and a further admission
with over activity, irritability and grandiose. What is the single most likely diagnosis?

a. Post partum blues


b. Post partum depression
c. Bipolar affective disorder
d. Recurrent depressive disorder
e. Schizo-affective disorder
Ans. The key is C. Bipolar affective disorder. [The patient has now features of depression
and 12 years ago she had similar symptoms and later symptoms of mania. So she is
suffering from bipolar affective disorder].

Mock-june 2013. 14. A 47-year-old man has diurnal mood variation, early morning
awakening and loss of interest and negative thoughts for four weeks. He has had no such
previous episodes, he has been made redundant recently and as a result his house has been
repossessed. What is the single most likely diagnosis?

a. Anxiety depressive disorder


b. Bipolar affective disorder
c. Major depressive disorder
d. Depression with personality disorder
e. Depression with psychotic symptoms

Ans. The key is C. Major depressive disorder.

Mock-june 2013. 15. A 32-year-old woman has poor concentration and forgetfulness for
four weeks. She feels sad and tired with insomnia and poor appetite. What is the single
most likely diagnosis?

a. Post partum blue


b. Post partum depression
c. Post partum psychosis
d. Recurrent depressive disorder
e. Depressive pseudodementia

Ans. The key is E. Depressive pseudodementia. [Depressive pseudodementia is a term


commonly used to describe a condition whereby a patient experiences a cognitive deficit
secondary to a primary mood disorder].

Mock-june 2013. 16. A 29-year-old woman has had delusion of poverty and guilty ideas for
some time. She has not had any such previous episodes. She is convinced that spies are
after her and trying to kill her. She also admits to feeling lethargic. What is the single most
likely diagnosis?

a. Anxiety depressive disorder


b. Bipolar affective disorder
c. Depressive personality disorder
d. Post partum blues
Ans. C. Depressive personality disorder. [Depression, and blaming and derogatory towards
self, feeling guilty and remorseful along with persecutory delusion (which makes it paranoid
type personality disorder) are suggestive of depressive personality disorder].

Mock-june 2013. 17. A 30-year-old lady has developed breast abscess 2 days after delivery
of her second baby. What is the single most likely causative organism?

a. E. Coli
b. Group B beta hemolytic Streptococcus
c. Staphylococcus albus
d. Staphylococcus feacalis
e. Staphylococcus aureus

Ans. E. Staphylococcus aureus.

Mock-june 2013. 18. A 28-year-old lady developed her baby after 10 days feels that she is
not good enough to take care of the baby and has a low mood, not eating well, exhaustion,
inability to enjoy things that she previously enjoyed and sleep disturbance.

a. Post partum depression


b. Post partum psychosis
c. Major depressive disorder
d. Depressive personality disorder
e. Depression with psychotic symptoms

Ans. A. Post partum depression [If symptoms last longer then 2 weeks from birth or start
later, patient could have postnatal depression. Postnatal depression can start any time in
the first year after giving birth.
Signs that you or someone you know might be depressed include:
a persistent feeling of sadness and low mood
lack of enjoyment and loss of interest in the wider world
lack of energy and feeling tired all the time
trouble sleeping at night and feeling sleepy during the day
difficulty bonding with your baby
withdrawing from contact with other people
problems concentrating and making decisions
frightening thoughts – for example, about hurting your baby].

Mock-june 2013. 19. A 57-year-old man recently underwent prostatectomy. Seven days
later he develops pain and swelling of his left leg. This was later confirmed as deep venous
thrombosis and he has to be started on warfarine. What is the optimal INR for this patient?

a. <1
b. 1–2
c. 2–3
d. 3–5
e. >5

Ans. C. 2 – 3

Mock-june 2013. 20. A 2-year-old boy has been unable to bear weight since the fall from a
chair the previous night. He is tender in the mid tibia but there is no obvious deformity.
What is the single most likely radiographic appearance?

a. Angulated fracture
b. Compound fracture
c. Epiphysial fracture
d. No trauma likely
e. Spiral fracture

Ans. E. Spiral fracture

Mock-june 2013. 21. An 8-year-old girl had recurrent central abdominal pains since 6
months. She also has occasional headaches. She maintains a normal appetite. There are no
abdominal signs. FBC, urinalysis and stool microscopy are normal. What is the single most
appropriate management?
a. Proton pump inhibitor
b. Pyeloplasty (+/-percutaneous nephrostomy)
c. Triple therapy
d. Reassurance
e. Sulphasalazine

Ans. D. Reassurance. [Given case is a case of abdominal migraine for which no specific
treatment is recommended. Abdominal migraine is believed to be a variant of migraine that
is common in children but rare in adults. Abdominal migraine is characterized by pain in the
center of the abdomen that may be severe. Symptoms can last for one hour or up to a
several days. Nausea and vomiting may be associated with the pain. Abdominal migraine is
treated by explanation and reassurance].

Mock-june 2013. 22. An 8-year-old boy has symptoms of right sided abdominal pain. He has
had symptoms of UTI in the past. Abdominal ultrasound shows right-sided hydronephrosis
and excretory isotope scan confirms PUJ obstruction. What is the single most appropriate
management?
a. Pancreatic enzyme
b. Metoclopramide
c. Pyeloplasty (+/- percutaneous nephrostomy)
d. Enema and laxatives
E. Aspirin

Ans. C. Pyeloplasty (+/- percutaneous nephrostomy). [As there is obstruction of pelvi-


ureteric junction it lead to hydronephrosis and it may need to relieve acute pressure by
percutaneous nephrostomy. For cure we have to go through the procedure of pyeloplasty].

Mock-june 2013. 23. A 14-year-old boy has had recurrent abdominal pains, malaise and
weight loss over 6 months. He has had a vague mass in the right iliac fossa. Colonoscopy
and biopsy of the caecal area shows transmural inflammation and granulomata. What is the
single most appropriate management?
a. Sulphasalazine
b. Appendicectomy
c. Aspirin
d. Enema and laxatives
e. Gluten free diet

Ans. A. Salphasalazine. [Recurrent abdominal pains, weight loss and transmural


inflammation and granulomata points towards the dx of Crohn’s disease which is treated
with sulphasalazine].

Mock-june 2013. 24. An 8-year-old has had recurrent abdominal pain for 2 months. His
appetite is reduced and he has faecal soiling. There are pitting masses throughout the
abdomen. He has a dilated rectum. What is the single most appropriate management?
a. Proton pump inhibitor
b. Pyeloplasty (+/- percutaneous nephrostomy)
c. Enema and laxative
d. Aspirin
e. Triple therapy

Ans. Enema and laxative. [Pitting masses throughout the abdomen with dilated rectum
indicates retained faecal matter. Enema and laxative are the treatment].

Mock-june 2013. 25. A 12-year-old has recurrent abdominal pain which is worse when he is
hungry. The pain is relieved by food or meal. He is tender in the epigastrium. Test has
revealed Helicobacter pylori infection. What is the single most appropriate management?
a. Proton pump inhibitor
b. Enema and laxative
c. Aspirin
d. Gluten free diet
e. Metoclopramide
Ans. A. Proton pump inhibitor. [As patient has H. pylori infection the most appropriate
management is triple therapy. But as it is not in the given options Proton pump inhibitor is
the next answer]. X
26 A, 27 D, 28 B, 29 C, 30 B

Mock-June 2013: 26. A 26-year-old man presents to the A & E department with shortness of
breath of 3 days duration. He has been coughing during the night and using his salbutamol
inhaler more frequently. What is the single most appropriate investigation?
a. Arterial blood gases
b. Chest X-ray
c. Electrocardiogram (ECG)
d. Haemoglobin
e. Magnetic resonance imaging scan (MRI) of the chest

Ans. A. Arterial blood gases. [Arterial blood gases are valuable for assessing severity of
exacerbations and following response to treatment. It is useful to differentiate between type1
and type 2 respiratory failure]. X

Mock-June 2013: 27. A 22-year-old man presents to the A & E with sudden onset of sharp right-
sided chest pain and shortness of breath. There is reduced air entry in the right side of the
chest.
What is the single most appropriate investigation?
a. Peak expiratory flow
b. Pulse oxymetry
c. Spiral computed Tomography scan of the chest
d. Chest X-ray
e. Arterial blood gases

Ans. D. Chest X-ray. [Features (sudden chest pain with shortness of breath, reduced air entry)
are consistent with pneumothorax for which Chest X-ray is the investigation of choice]. X

Mock-June 2013: 28. A 78-year-old woman is brought by her family to the A & E department
with shortness of breath and occasional cough over the previous 72 hours. There is a coarse
crepitation present at the right lung base. What is the single most appropriate investigation?
a. Arterial blood gases
b. Chest X-ray
c. Electrocardiogram (ECG)
d. Haemoglobin
e. Magnetic resonance imaging scan (MRI) of the chest.

Ans. B. Chest X-ray. [Probable pneumonia. Chest X-ray should be done]. X


Mock-June 2013: 29. A 26-year-old woman who has returned from a holyday in America
develops sudden left sided chest pain and breathlessness. Her chest is clear. What is the single
most appropriate investigation?
a. Peak expiratory flow
b. Pulse oxymetry
c. Spiral computed tomography (CT) scan pulmonary angiogram of the chest
d. Chest X-ray
e. Arterial blood gases

Ans. C. Spiral computed tomography (CT) scan pulmonary angiogram of the chest. [Probable
pulmonary embolism which is a well-known complication of prolonged air travel and Spiral
computed tomography (CT) scan pulmonary angiogram of the chest is done to establish the
diagnosis]. X

Mock-June 2013: 30. A 23-year-old man with type 1 diabetes presents with 48 hours of
increasing confusion and breathlessness. His chest is clear and his blood glucose concentration
is 28 mmol/L. What is the single most appropriate investigation?
a. Chest X-ray
b. Arterial blood gas
c. Electrocardiogram (ECG)
d. Haemoglobn
e. Magnetic resonance imaging (MRI) scan of the chest.

Ans. B. Arterial blood gases. [Probabe DKA. In patients with DKA, arterial blood gases (ABGs)
frequently show typical manifestations of metabolic acidosis, low bicarbonate, and low pH (less
than 7.3)]. X

Mock-June 2013: 31. A 55-year-old man has weight loss, dyspnea and syncope. He smokes 20
cigarettes a day. Investigation confirms squamous cell carcinoma in the left bronchus. What is
the single most important likely biochemical abnormality to be associated with the condition?
a. Hyperkalaemia
b. Hypercalcaemia
c. Hypernatremia
d. Hypocalcemia
e. Hypomagnesemia

Ans. B. Hypercalcaemia. [The main pathogenesis of hypercalcemia in malignancy is increased


osteoclastic bone resorption. Enhanced bone resorption in malignancy is mainly secondary to
different humoral factors released by tumour cells locally or systemically. The main humoral
factor is parathyroid hormone related protein produced by many solid tumours. It increases
calcium by activating parathyroid hormone receptors in tissue, which result in osteoclastic bone
resorption. It also increases renal tubular resorption of calcium]. X
Mock-June 2013: 32. A 21-year-old woman in her 1st pregnancy has been admitted at 39 weeks
gestation with breech presentation. She has had an eclamptic fit at home. She is conscious and
observations are stable. What is the single most appropriate IV therapy to prevent further fits?
a. Chlomethiazole
b. Diazepam
c. Labetalol
d. Magnesium sulphate
e. Phenytoin

Ans. D. Magnesium sulphate. [Magnesium sulphate should be administered as infusions. If fits


occurs loading dose of magnesium sulphate should be given. If fits recur magnesium sulphate
bolus should be given till fits are controlled or side effects are evident (loss of reflexes or
respiratory rate <14/min)]. X

Mock-June 2013: 33. A 14-year-old girl developed an itchy, scaly patch on her scalp. She has a
similar patch that cleared spontaneously 2 years ago. Her aunt had similar undiagnosed rash on
the extensor aspects of her elbow and knees. What is the single most likely diagnosis?
a. Eczema
b. Fungal infection
c. Impetigo
d. Lichen sclerosis
e. Psoriasis

Ans. E. Psoriasis. [Chronic plaque psoriasis is typified by itchy, well-demarcated circular-to-oval


bright red/pink elevated lesions (plaques) with overlying white or silvery scale, distributed
symmetrically over extensor body surfaces and the scalp].

Mock-June 2013: 34. A 58 year old man suddenly becomes shocked several days after suffering
an acute myocardial infarction. His chest X-ray shows a large globular shaped heart and clear
lung fields. What is the single most likely diagnosis.
a. Acute pericarditis
b. Atrial thrombus
c. Cardiac tamponade
d. Heart block
e. Left ventricular aneurism

Ans. C. Cardiac tamponade. [Large globular heart indicates cardiac tamponade. Shock is due to
compromised cardiac output due to tamponade]. X

Mock-June 2013: 35. A 63-year-old man continues to experience chest pain and has mild
pyrexia two days after an acute myocardial infarction. His ECG shows widespread ST elevation
and upward concavity. What is the single most likely diagnosis?
a. Left ventricular endocardial thrombus
b. Post myocardial infarction (Dressler’s syndrome)
c. Acute pericarditis
d. Ruptured inerventricular septum
e. Ruptured papillary muscle

Ans. C. Acute pericarditis. [Chest pain, mild pyrexia and typical ECG findings (widespread ST
elevation, upward concavity) indicates pericarditis. Dressler syndrome has similar pictute but
occurs after 2-3 weeks of MI, not too early like 2-3 days]. X

Mock-june 2013: 36. A 48-year-old man has continuous anterior chest pain, worse on
inspiration and pyrexia four weeks after a myocardial infarction. His ESR is raised. What is the
single most likely diagnosis?
a. Left ventricular endocardial thrombus
b. Post myocardial infarction (Dressler’s Syndrome)
c. Acute pericarditis
d. Ruptured intraventricular septum
e. Ruptured papillary muscle

Ans. B. Post myocardial infarction (Dressler’s Syndrome). [Usually occurs 2-5 weeks after the
initial event but it can be delayed for as long as 3 months. It is characterised by pleuritic chest
pain, low-grade fever and pericarditis, which may be accompanied by pericardial effusion. It
tends to follow a benign clinical course (though it may suggest further infarction). It is thought
to be immune-mediated (antiheart antibodies may be present)]. X

Mock-june 2013: 37. A 60-year-old woman suddenly develops severe left ventricular failure
following a myocardial infarction. Her echocardiogram demonstrates mild regurgitation. What
is the single most likely diagnosis?
a. Left ventricular endocardial thrombus
b. Post myocardial infarction (Dressler’s syndrome)
c. Acute pericarditis
d. Ruptured intraventricular septum
e. Ruptured papillary muscle

Ans. E. Ruptured papillary muscle. [In ruptured papillary muscle there occurs mitral
regurgitation]. X

Mock-june 2013: 38. A 55-year-old man returns for routine follow up 6 weeks after myocardial
infarction. He gets breathless when walking uphill. His ECG shows ST elevation in the anterior
chest leads. What is the single most likely complication?
a. Left ventricular endocardial thrombus
b. Post myocardial infarction (Dressler’s syndrome)
c. Acute pericarditis
d. Ruptured intraventricular septum
e. Left ventricular aneurysm
Ans. E. Left ventricular aneurysm. [Left ventricular aneurysm may occur following a heart
attack. They usually arise from a patch of weakened tissue in a ventricular wall, which swells
into a bubble filled with blood. This, in turn, may block the passageways leading out of the
heart, leading to severely constricted blood flow to the body lowering stamina. Left ventricular
aneurysm is frequently associated with ST elevation]. X

Mock-june 2013: 39. A 70-year-old man is brought to the A & E department by his daughter
following overdose of 10 paracetamol tablets. His wife died 2 weeks ago. His only relevant past
medical history was treatment for depression 10 years ago. His paracetamol level is below the
treatment level. What is the most appropriate initial management?
a. Referral for social worker
b. Discharge home with advice
c. Refer for urgent psychiatric opinion
d. Refer for clinical psychiatric opinion

Ans. C. Refer for urgent psychiatric opinion. [Urgent psychiatric opinion is needed in psychiatric
emergency where the person (patient himself) or person surrounding him is endangered by life
threatening or destructive acts. In this patient if urgent intervention is not taken may overdose
can happen again]. X

Mock-june 2013: 40. A 24-year-old woman fell and injured her right knee. The knee is swollen
and tender to valgus stressing. Which of the following ligament is the most likely to have been
damaged?
a. Anterior cruciate
b. Lateral collateral
c. Medial collateral
d. Oblique popliteal
e. Posterior

Ans. C. Medial collateral. [Positive valgus stress test indicate medial collateral rupture]. X

Mock-june 2013: 41. A 22-year-old man has had acute painful, red eye with blurring of vision
for one day. He had similar episodes one year ago and has had back pain and stiffness relieved
by exercise and diclofenac for years. What is the single most likely cause of the red eye?
a. Retinitis
b. Conjunctivitis
c. Iritis
d. Scleritis
e. Glaucoma

Ans. C. Iritis. [The young man has suggestive features of ankylosing spondylitis where iritis is a
common association]. X
Mock-june 2013: 42. A 70-year-old man has back pain, anemia and weight loss of gradual onset.
In addition to renal failure, he has hypercalcemia. What is the most appropriate investigation?
a. ANCA
b. Bone marrow aspirate
c. Creatinine clearance
d. Culture of MSU specimen
e. Cystoscopy

Ans. B. Bonemarrow aspirate. [Features are suggestive of multiple myeloma. Bone marrow
aspirate will show plasma cell infiltrate (60% or more plasma cells in bone marrow)]. X

Mock-june 2013: 43. A 55-year-old man has leg swelling. He has heavy proteinuria and his
serum creatinine concentration is 185mmol/L. What is the single most appropriate
investigation?
a. IVU
b. Nuclear scan
c. Renal biopsy
d. Renal Ultrasound
e. Serum cholesterol concentration

Ans. C. Renal biopsy. [At this age the most common cause of nephrotic syndrome is focal
glomerulosclerosis and then membranous glomerulonephritis for both of which renal biopsy is
needed to establish the diagnosis]. X

Mock-june 2013: 44. A 75-year-old man has had a frequency of micturition, poor urinary stream
and some incontinence for 2 years. His serum creatinine concentration is 350mmol/L. What is
the single most appropriate investigation?
a. ANCA
b. Bone marrow aspirate
c. Creatinine clearance
d. Culture of MSU specimen
e. Cystoscopy
f. US of the kidney

Ans. F. US of the kidney. [features of prostatism is suggestive of prostate pathology and from
the given options most appropriate is F. US of the kidney. Please note though US kidney is
mentioned it is actually intended to mention US of KUB!! Diagnosis is BPH and US will show
obstructive uropathy (like dilatation of ureter and calyx or hydronephrosis)]. X

Mock-june 2013: 45. A 65-year-old woman has urinary frequency, weight loss, hematuria and
blood clots. An abdominal ultrasound is inconclusive. What is the single most appropriate
investigation?
a. IVU
b. Nuclear scan
c. Renal Ultrasound
d. Serum cholesterol concentration
e. Cystoscopy

Ans. E. Cystoscopy. [Features are of bladder cancer supported by age, weight loss and
hematuria. In bladder cancer there develop UTI like symptom as frequency, dysuria etc]. X

Mock-june 2013: 46. A 20-year-old woman with multiple sclerosis presents with blurring of
vision and pain in the eye on movement. Where is the lesion?
a. Cerebellum
b. Optic discs
c. Optic nerve
d. Occipital
e. Retina

Ans. The key is C. Optic nerve. [Optic neuritis is a well known feature of multiple sclerosis.
Described lesion is optic neuritis! Optic neuritis is a demyelinating inflammation of the optic
nerve. It is also known as optic papillitis (when the head of the optic nerve is involved) and
retrobulbar neuritis (when the posterior of the nerve is involved)]. X

Mock-june 2013: 47. A 65-year-old man after an open cholecystectomy experiences numbness
around the umbilicus. Which nerve is damaged?
a. Diaphragmatic nerve
b. T4 nerve
c. Intercostal nerve
d. L1-L2 nerve root
e. T10 nerve root

Ans. The key is E. T10 nerve root. [Dermatom pattern of T10 nerve: Traverses posteriorly from
T10 to anteriorly through the umbilicus].

Mock-june 2013: 48. A 30-year-old lady presents with sudden onset of right sided ilac fossa
pain. She also experiences shoulder tip pain. What is the single most likely nerve damage?
a. Phrenic nerve
b. T12 nerve root
c. T5 nerve root
d. T10 nerve root
e. Intercostal nerve

Ans. The key is A. Phrenic nerve. [Phrenic nerve can be justified by referred pain to right
shoulder but relation of right iliac fossa pain and phrenic nerve couldn’t be established!!] X
Mock-june 2013: 49. A 25-year-old woman is 8 weeks pregnant. She has experienced a degree
of vaginal bleeding and severe left sided abdominal pain. Her pulse rate is 100 beats/minute
and her BP is 110/75 mmHg. There is marked cervical excitation on vaginal examination. What
is the single most appropriate management?
a. Administration of anti-D
b. Bimanual vaginal examination
c. Blood transfusion
d. Colposcopy
e. Culdocentasis

Ans. The key is E. Culdocentasis. [In given case diagnostic laparoscopy should be the answer but
as it is not in the options we should go for the most appropriate investigation from the given
options which is E. Culdocentasis here. As patient is not in shock the dx is not ruptured ectopic
pregnancy yet but leaking ectopic pregnancy in which there occurs accumulation of blood in
pouch of douglas which can be revealed by culdocentasis]. X

Mock-june 2013: 50. An 18-year-old woman who is 13 weeks pregnant has a single episode of
painless postcoital bleeding. The fetal heart is audible during the dopplar scan. What is the
single most appropriate management?
a. Evacuation of retained products
b. Group and save serum
c. Laparoscopy
d. Monitor
e. Oral stilbesterol

Ans. The key is D. Monitor. [Painless postcoital bleeding gives rise to suspicion of placenta
previa (cervical ectropion should be kept in mind also) for which at this stage we should
monitor the patient]. X

Mock-june 2013: 51. A 28-year-old woman who is 10 weeks pregnant has had vaginal bleeding
and period like pain for 5 days. Over the next 48 hours the symptoms have decreased.
Examination is inconclusive. Beta HCG is very low and US scan reveals minimal uterine contents.
What is the single most appropriate management?
a. Speculum examination
b. Ultrasound scan
c. Blood transfusion
d. Colposcopy
e. Evacuate product of conception

Ans. The key is E. Evacuate product of conception. [Diagnosis is incomplete abortion.


Management is evacuation of product of conception]. X
Mock-june 2013: 52. A 28-year-old woman who is 9 weeks pregnant has a small amount of
spontaneous vaginal bleeding. She has a mild vague lower abdominal pain. Her uterus is non-
tender and equivalent for dates. What is the single most appropriate management?
a. Administration of Anti-D
b. Bi-manual vaginal examination
c. Blood transfusion
d. Culdocentesis
e. Transvaginal USS

Ans. The key is E. Transvaginal USS. [As the uterus is equivalent for dates it is likely threatened
abortion and transvaginal USS should be done to rule out complete or incomplete miscarriage
and to see fetal heart beats]. X

Mock-june 2013: 53. A woman who is 10 weeks pregnant has heavy vaginal bleeding with
marked uterine pain. Her pulse rate is 110 beats/min and BP is 110/75mmHg. Vaginal
examination confirms that the blood is uterine and her cervix has begun to dilate.
a. Evacuation of retained products
b. Group and save serum
c. Laparoscopy
d. Monitor
e. Oral stilbesterol

Ans. The key is B. Group and save serum. [The diagnosis is inevitable abortion. 1 st choice of
management according to NICE is natural. So before taking further decision we can do blood
grouping and save serum for the reference for next intervention]. X

Mock-june 2013: 54. A 23-year-old lady with multiple sclerosis presents with ataxia, dysarthria
and nystagmus. Which of the following is the most likely location of the lesion?
a. Spinal cord
b. Brain stem
c. Cerebellum
d. Cerebral cortex
e. Medulla oblongata

Ans. The key is C. Cerebellum. [Ataxia, dysarthria and nystagmus are well known features of
cerebellar lesion]. X

Mock-june 2013: 55. A 45-year-old man presents with headache and vomiting. He is diagnosed
with meningitis and a decition to perform a lumber puncture has been made. Which of the
following structure will be pierced just before reaching the desired space?
a. Dura mater
b. Pia mater
c. Skin
d. Longitudinal muscles
e. Arachnoid matter

Ans. The key is E. Arachnoid matter. [The CSF occupies the subarachnoid space (between the
arachnoid mater and the pia mater). So just before reaching this subarachnoid space we have
to pierce the arachnoid matter]. X

Mock-june 2013: 56. A 40-year-old man presents with headache, vomiting and fever. He has
been diagnosed with meningitis. Which of the following is a correct landmark to perform a
lumber puncture?
a. L4
b. Iliac crest
c. Superior anterior iliac spine
d. Cox cocedeus
e. L5 vertebrea

Ans. The key is B. Iliac crest. [Lumber puncture is done in the L3-L4 interspace by palpating the
right and left posterior superior spines of iliac crest and moving the fingers medially towards
the spine]. X

Mock-june 2013: 57. A man was involved in a fight and sustained a blow to his mastoid bone.
Which one is fractured?
a. Sphenoid bone
b. Mandibular bone
c. Temporal bone
d. Parietal bone
e. Occipital bone

Ans. The key is C. Temporal bone. [The temporal bone consists of four parts— the squamous,
mastoid, petrous and tympanic parts]. X

Mock-june 2013: 58. A 35-year-old man is being investigated for infertility. Which of the
following tests would prove that he has complete azoospermia?
a. Prolactin
b. Oestrogen level
c. Testosterone
d. Gonadotrophine
e. Follicle stimulating hormone

Ans. The key is C. Testosterone. [Low testosterone may lead to azoospermia. But surprisingly
testosterone replacement can improve libido but not help azoospermia as it causes follicle
stimulating hormone suppression. So testosterone rise should be done by natural means to
improve fertility!] X
Mock-june 2013: 59. A 70-year-old lady brings her husband who is 83 years of age. Which of the
following additional factor will confirm the diagnosis?
a. Progressive functional disorder
b. Fluctuating functional disorder
c. Incontinence of urine
d. Suicidal intension
e. Stepwise deterioration

Ans. The key is A. Progressive functional disorder. [a. Alzheimer, b. Lewy body dementia, c.
Normal pressure hydrocephalus, e. Vascular dementia]. X

Mock-june 2013: 60. A 5-year-old girl has had an upper respiratory tract infection (URTI) for 3
days and has been treated with paracetamol by her mother. The last 12 hours, she has been
hot and irritable with severe pain in the right ear.
a. Acoustic neuroma
b. Chronic serous otitis media (glue ear)
c. Otitis media
d. Foreign body
e. Herpes zoster infection

Ans. The key is C. Otitis media. X

Mock-june 2013: 61. A 38-year-old woman has just returned from a holyday where he went
swimming every day. For the last few days he has irritation in both ears. Now his right ear is
hot, red, swollen and acutely painful. What is the most likely diagnosis?
a. Impacted wax
b. Mumps
c. Otitis media
d. Otitis externa
e. Perforation of ear drum.

Ans. D. Otitis externa. [Presentation may prompt one to think otitis media, but in swimming
there is only exposure of external ear to the water and no other risk factor for otitis media!! So
it is not otitis media but otitis externa]. X

Mock-june 2013: 62. A 45-year-old man has a smooth, tender swelling extending from the ear
to the angle of the jaw of sudden onset. He has a temperature of 38.5 C. What is the single
most likely diagnosis?
a. Presbycusis
b. Temporomandibular joint pain
c. Trigeminal neuralgia
d. Mumps
e. Otitis medisa
Ans. D. Mumps. X

Mock-june 2013: 63. An 18-year-old man has a smooth, tender swelling from the ear to the
angle of the jaw of sudden onset. He has a temperature of 38.5 C. What is the single most likely
diagnosis?
a. Presbycusis
b. Temporomandibular joint pain
c. Trigeminal neuralgia
d. Mumps
e. Otitis media

Ans. D. Mumps. X

Mock-june 2013: 64. A 75-year-old woman has weakness of left side of her face. She has had a
painful ear for 48 hours. There are pastules in the left ear canal and on the eardrum. What is
the single most likely diagnosis?
a. Impacted ear wax
b. Mumps
c. Otitis externa
d. Otitis media
e. Perforation of the eardrum
f. Ramsey Hunt syndrome

Ans. F. Ramsey Hunt Syndrome. [Herpes zoster oticus when presents with facial nerve
involvement is known as Ramsey Hunt syndrome].

Mock-june 2013: 65. A 45-year-old woman has her left ear syringed 48 hours ago. During this
procedure she had acute pain. Now she has discharge and deafness. Which vein is likely to be
involved?
a. Perforated eardrum
b. Azygous vein
c. Bracheocephalic vein
d. Cephalic vein
e. External iliac vein

Ans. Perforated eardrum!! What type of question is it and what type of vein is it I don’t know!!!
X

Mock-june 2013: 66. A 43-year-old has pain and stiffness to her leg. The limb is swollen and
cyanosed from the upper thigh to her ankle. A large uterine mass is palpable. Which vein is
likely to be involved?
a. Inferior vena cava
b. Internal iliac vein
c. External iliac vein
d. Long sephanous vein
e. Median cubital vein

Ans. C. External iliac vein. [external iliac vein is the deep vein of lower limb and compression of
it can result in chronic leg pain and swelling along with cyanosis]. X

Mock-june 2013: 67. A 41-year-old woman has varicose vein on the lateral side of her right
lower leg. The vein is tortuous and dilated. Which vein is likely to be involved?
a. Popliteal vein
b. Profunda femoris vein
c. Renal vein
d. Short sephanous vein
e. Axillary vein

Ans. D. Short sephanous vein. [Short sephanous vein courses through lateral side of the leg
while long or great sephanous vein courses medially]. X

Mock-june 2013: 68. A 25-year-old man has a varicosity on his lower abdominal wall. There is a
small testicular mass which is subsequently found to be malignant. Which vein is likely to be
involved?
a. Axillary vein
b. Azygous vein
c. Bracheocephalic vein
d. Renal vein
e. External iliac vein

Ans. D. Renal vein. X

Mock-june 2013: 69. A 60-year-old has pain and stiffness of his left leg. His calf is swollen and
feels hard. Which vein is likely to be involved?
a. Inferior vena cava
b. Internal iliac vein
c. Internal jugular vein
d. Long sephanous vein
e. Popliteal vein

Ans E. popliteal vein. X

Mock-june 2013: 70. A 28-year-old man who has spent his weekend decorating his house has
swelling and discomfort in his right arm of sudden onset. The upper limb is swollen, congested
and cyanosed. Which vein is involved?
a. Popliteal vein
b. Profunda femoris vein
c. Renal vein
d. Short sephanous vein
e. Axillary vein

Ans. E. Axillary vein. X

Mock-june 2013: 71. A 30-year-old lady presents with foul smelling discharge. Endocervical
swab shows trichomonas. Which of the following is the most appropriate treatment?
a. Amoxicillin
b. Metronidazole
c. C0-trimoxazole
d. Tetracycline
e. Ceftriaxone

Ans. B. Metronidazole. [Metronidazole is very effective if taken correctly. One should have to
take metronidazole twice a day, for five to seven days. Ref: NHS]. X

Mock-june 2013: 72. A 69-year-old man with heart failure develops gout. He has recently been
started on treatment and his heart failure worsens. Which of the following is the most likely
cause?
a. NSAIDs
b. B-blockers
c. Bendroflumethiazide
d. Calcium channel blocker
e. Allopurinol

Ans. A. NSAIDs. [NSAIDs used for gout worsens heart failure by retention of salt and water]. X

Mock-june 2013: 73. A 7-year-old 7 days post tonsillectomy presents with nose bleed. He is
currently not bleeding. What is the most appropriate treatment?
a. Apply pressure on the soft part
b. Antibiotics
c. Reassure
d. Surgical exploration
e. Local adrenaline

Ans. B. Antibiotics. [Bleeding from post operative site may occur from infection which causes
necrosis and sloughing out of dead tissue. So such bleeding is treated by antibiotics to control
risk of further bleeding]. X

Mock-june 2013: 74. A 3-year-old boy has a sudden onset of cough, shortness of breath and
noisy breathing. His mother reports that this is followed by a choking episode while he was
playing.
a. Cystic fibrosis
b. Epiglottitis
c. Immunodeficiency
d. Inhaled foreign body
e. Recurrent aspiration

Ans. D. Inhaled foreign body. [During playing with toy, toy or part of it can enter respiratory
tract of young children and cause above mentioned symptoms]. X

Mock-june 2013: 75. A 2-year-old boy has repeated chest infections, offensive loose stools and
recurrent rectal prolapse.
a. Tracheomalacia
b. Tuberculosis
c. Asthma
d. Cystic fibrosis
e. Bronchiectasis

Ans. D. Cystic fibrosis. [Repeated chest infections, offensive stools, (stress incontinence) and
rectal prolapse are recognized features of cystic fibrosis]. X

Mock-june 2013: 76. A 4-year-old girl who has had a runny nose for 12 hours has woken up
2’oclock in the morning with difficulty in breathing, a barking cough. What is the single most
likely diagnosis?
a. Asthma
b. Bronchiectasis
c. Bronchiolitis
d. Bronchopulmonary dysplasia
e. Croup

Ans. E. Croup. [Age, runny nose, dyspnea, and barking cough are suggestive features of croup].
X

Mock-june 2013. 77. A 4-year-old girl whose parent have refused to allow immunizations has a
fever, difficulty in breathing, stridor, drooling and central cyanosis. What is the single most
likely diagnosis?
a. Cystic fibrosis
b. Epiglottitis
c. Immunodeficiency
d. Inhaled foreign body
e. Recurrent aspiration

Ans. B. Epiglottitis. [Haemophilus influenza, Haemophilus parainfluenzae, Streptococcus


pneumoniae, and group A streptococci are the common causes of epiglottitis]. X

Mock-june 2013: 78. A 16-year-old footballer presents with sudden onset testicular pain, which
started an hour ago. What is the most likely diagnosis?
a. Trauma
b. Testicular torsion
c. Epididymo-orchitis
d. Varicocele
e. Hernia

Ans. B. Testicuar torsion. [Sudden onset, no fever is more likely to be testicular torsion]. X

Mock-june 2013: 79. A 70-year-old man who is on bendroflumethiazide develops an acute


swelling of his right knee. What is the single most appropriate diagnostic investigation?
a. Serum uric acid
b. Reactive protein
c. D-dimer
d. Blood culture
e. Plain X-ray of the knee

Ans. A. Serum uric acid. [Bendroflumethiazide can cause hyperurecemia leading to gout]. X

Mock-june 2013: 80. A 45-year-old man with diabetes mellitus has a high ferritin. What type of
cancer is the most likely to develop?
a. Liver cancer
b. Lung cancer
c. Renal cancer
d. Colonic cancer
e. Gastric cancer

Ans. A. Liver cancer. [In patient with DM ferritin level is higher than control. This ferritin is
indicative of high iron level in liver ultimately leading to liver cancer]. X

Mock-june 2013: 81. A 70-year-old man had a painful swelling of his right testis for 36 hours. He
has a history of dysuria and frequency of micturition and has a temperature of 38.C
a. Epididymal cyst
b. Epididymo-orchitis
c. Hydrocele
d. Inguinal hernia
e. Scrotal abscess

Ans. B. Epididymo-orchitis. [Painful swelling of testis with features of UTI and raised
temperature makes the likely diagnosis to be Epididymo-orchitis]. X

Mock-june 2013: 82. A 25-year-old man has had painless enlargement of his left testis for 3
months. The left testis is 3 times the size of right testis and is non-tender. What is the single
most likely diagnosis?
a. Sebacious cyst of the scrotunm
b. Testicular hematoma
c. Testicular neoplasm
d. Hydrocele
e. Epididymal cyst

Ans. D. Hydrocele. [Though testicular neoplasm is also possible but hydrocele is much more
common than tumour].

Mok-june 2013: 83. A 17-year-old lady presents to the A&E department after taking 45 tablets
of paracetamol following an argument with her boyfriend. The incident happened 10 hours ago.
What is the single most appropriate next action?
a. Activated charcoal
b. Check paracetamol level
c. N-acetylcysteine
d. Observe
e. Do blood tests after 4 hours

Ans. C. N-acetylcisteine. [If >8–24h and suspicion of large overdose (>7.5g) err on the side of
caution and start N-acetylcysteine, stopping it if level below treatment line and INR/ALT
normal]. Ref: OHCM, 9th edition, page-856. X

Mock-june 2013: 84. A 56-year-old man with known HIV presents with chronic diarrhea. What
is the most likely cause?
a. Cryptococcus infection
b. Toxoplasmosis
c. Pneumocystis carinii
d. Cryptosporidium
e. Herpes zoster

Ans. D. Cryptosporidium. [In AIDS patients common cause of diarrhea is cryptosporidium]. X

Mock-june 2013: 85. A 65-year-old man who is an intravenous drug abuser presents with
weight loss and maculopapular rash. What is the single most likely cause?
a. Cryptococcus infection
b. HHV8
c. Pneumocystis carinii
d. Human papilloma virus
e. Cryptosporidium

Ans. B. HHV8. [human herpes virus 8 or, kaposis sarcoma associated herpes virus]. X

Mock-june 2013: 86. A 42-year-old woman has not had a period for 10 months. She feels well
but she has hot flushes. Her mother also had an early menopause.
a. Cervical smear
b. Chest-x-ray
c. Dexa scan
d. FSH/LH

Ans. D. FSH/LH. [FSH and LH are raised in premature ovarian failure; an FSH level ≥20 IU/l in a
woman aged under 40 with secondary amenorrhoea indicates premature ovarian failure]. X

Mock-june 2013: 87. A 60-year-old woman under long term follow up after a lumpectomy and
therapy for breast cancer at the age of 50 comes to discuss her bone health following a stress
fracture to her metatarsal bone.
a. Lumber spine X-ray
b. Dexa scan
c. Mid stream urine culture
d. No investigation
e. Serum calcium concentration

Ans. B. Dexa scan. [60 yrs of age female are prone to osteoporosis due to reduced oestrogen
and dexa-scan is used in susceptible patients]. X

Mock-june 2013: 88. A 36-year-old lady who is an intravenous drug user has had weight loss for
the past 4 months. He now presents with drowsiness and increasing confusion. What is the
single most likely cause?
a. Cryptococcus infection
b. Toxoplasmosis
c. Pneumocystis carinii
d. Cryptosporidium
e. Herpes zoster

Ans. The key is A. Cryptococcus infection. This is a wrong key!! Correct key is B. Toxoplasmosis.
[Case of HIV. Cryptococcus usually causes meningitis in AIDS patients and in meningitis mental
clearance is usually maintained! That is there does not occur confusion. On the other hand
cerebral toxoplasmosis is the infestation where drowsiness and confusion occurs]. X

Mock-june 2013: 89. A 36-year-old woman is complaining of mood swings, palpitations and
irregular menses.
a. Transvaginal USS
b. FSH
c. Chest X-ray
d. Dexa scan
e. ECG

Ans. B. FSH. [Symptoms are of perimenopause and in this patients likely indicates going to be
premature ovarian failure]. X
Mock-june 2013: 90. A 20-year-old mother brings her 2 year old child and she is worried that
her child be deaf as she is deaf herself. What is the single most appropriate investigation?
a. Audiometry
b. Tuning fork
c. Audiometry brain stimulating
d. Reassure
e. Distraction test
Ans. The key is A. Audiometry. X

Mock-june 2013: 91. A 75-year-old man is having a laparoscopic cholecystectomy. 2 ports on


the lateral abdomen have been inserted. When placing the 3rd port, which structure will be
damaged?
a. Linea alba
b. Latisimus dorsi
c. Pectolaris major
d. Skin
e. Intercostal muscle

Ans. The key is A. Linea alba. X

Mock-june 2013: 92. A 21-year-old student has several episodes of palpitations, sweating,
nausea and overwhelming fear of sudden onset. On one occasion, she has woken from sleep in
this state and fears she is going insane. There is no previous history of any psychiatric disorder.
What is the single most likely diagnosis?
a. Adjustment disorder
b. Agoraphobia
c. Anorexia nervosa
d. Bipolar disorder
e. Panic attack

Ans. The key is E. Panic attack. [Panic attack can be frightening and happen suddenly, often for
no clear reason with following features, palpitations, sweating, trembling, hyperventilation,
nausea, and few more]. X

Mock-june 2013: 93. A 40-year-old businessman is worried that his alcohol consumption has
reached dangerous levels. He has recently changed jobs, which involve travelling and corporate
hospitality. He finds it increasingly difficult to give presentations to new business associates and
finds that alcohol helps him to function.
a. Major depressive episode
b. Obsessive compulsive disorder
c. Panic disorder
d. Post traumatic stress disorder
e. Social anxiety disorder
Ans. The key is E. Social anxiety disorder. X

Mock-june 2013: 94. A 32-year-old mother is in tears, complaining of constant irritability with
her 2 small children and inability to relax. She describes herself as easily startled with poor
sleep disturbed by nightmares, following a serious house fire one year ago while the family
slept.
a. Post traumatic stress disorder
b. Adjustment disorder
c. Agoraphobia
d. Anorexia nervosa
e. Bipolar disorder

Ans. The key is A. Post traumatic stress disorder. [ Post-traumatic stress disorder (PTSD) is an
anxiety disorder caused by very stressful, frightening or distressing events. Someone with PTSD
often relives the traumatic event through nightmares and flashbacks. They may also have
problems sleeping, such as insomnia, and find concentrating difficult]. X

Mock=june 2013: 95. A 49-year-old man has returned home 6 times daily from work for the
past many years worried that he has left the taps running. His employers are concerned about
his poor work productivity.
a. Adjustment disorder
b. Agoraphobia
c. Anorexia nervosa
d. Obsessive compulsive disorder
e. Generalized anxiety disorder

Ans. The key is D. Obsessive compulsive disorder. [Obsessive-compulsive disorder (OCD) is


characterized by unreasonable thoughts and fears (obsessions) that lead one to do repetitive
behaviors
(compulsions)]. X

***Mock-june 2013: 96. A 40-year-old woman has acute symptoms of palpitations, sweating,
difficulty in breathing and feeling unsteady on her feet. She has noticed her symptoms occur
only in crowded public places.
a. Major depressive disorder
b. Obsessive compulsive disorder
c. Agoraphobia
d. Post traumatic stress disorder
e. Social anxiety disorder

Ans. C. Agoraphobia. [One may have agoraphobia if he may has fear of:
Entering shops, crowds, and public places.
Travelling in trains, buses, or planes.
Being on a bridge.
Being in a lift.
Being in a cinema, restaurant, etc, where there is no easy exit.
Being anywhere far from your home].

Mock-june 2013: 97. A 75-year-old man has an episode of syncope while working in the garden.
When visited by his doctor 30 minutes later, he is in sinus rhythm; his pulse rate is 74 bpm. His
blood pressure is 110/60 mmHg and he has a systolic murmur. What is the single investigation
most likely to provide the definitive diagnosis?
a. Coronary angiogram
b. Echocardiogram
c. Electrocardiogram
d. 24 hour ambulatory blood pressure
e. 24 hour taped ECG

Ans. B. Echocardiogram. [Probable diagnosis is aortic stenosis which can cause syncope during
exercise particularly in elderly person which can be diagnosed by echocardiogram].

Mock-june 2013: 98. An 83-year-old-woman who is resident in a nursing home is admitted to


hospital with a four day history of diarrhea. She has had no weight loss or change in appetite.
She has been on analgesics for 3 weeks for her back pain. She is in obvious discomfort. On
rectal examination, she is found to have fecal impaction. What is the single most appropriate
immediate management?

a. Codeine phosphate for pain relief


b. High fibre diet
c. Oral laxative
d. Phosphate enema
e. Urinary catheterization

Ans. The key is D. Phosphate enema. [This diarrhea is overflow diarrhea of fecal impaction and
phosphate enema will help to bring out feacal mass and resolve the condition].

Mock-june 2013. 99. A 65-year-old man has a productive cough and fever. He has influenza and
his temperature is 38.5 C. There is reduced air entry and crepitations bilaterally. What is the
single most likely cause of the pneumonia?

a. Haemophilas influenza
b. Klebsiella pneumonia
c. Mycoplasma pneumonia
d. Staphylococcus aureus
e. Streptococcus pneumonia
Ans. The key is D. Staphylococcus aureus. [Pneumonia following influenza is frequently due to
staphylococcus aureus with bilateral crepitations and with productive cough and fever].

Mock-june 2013. 100. A 67-year-old lady has lung cancer and she suffers from heartburn, which
increases at night. She has tooth carries and has offensive mouth odour. She can not cope with
it. What is the single most appropriate treatment for her?

a. Erythromycin
b. Vancomycin
c. Metronidazole
d. Proton pump inhibition
e. Anty H. pylori treatment

Ans. The key is C. Metronidazole. This is a wrong key. Correct key is D. Proton pump inhibitor.
[Cancer itself does not cause halitosis (though lung cancer can produce halitosis but not treated
by metronidazole unless there are signs of infection) but secondary to chemotherapy as its
side-effects. Metronidazole is not the treatment of chemotherapy halitosis. This patient has
clear picture of GERD (tooth carries are due to acid reflux) and it is a leading cause of halitosis.
Single most appropriate treatment for GERD induced halitosis is proton pump inhibitor].

Mock-june 2013. 101. A 20-year-old man has a head on collision in a car. He is not breathless
and has chest pain as he fractured his 5 – 7th ribs. Chest X-ray confirms this fractures. What is
the most appropriate initial action?

a. Antibiotics
b. Analgesia
c. Oxygen mask
d. Physiotherapy
e. Refer to surgeon

Ans. The key is B. Analgesia. [Most appropriate initial action in flial chest is oxygen mask if the
patient is breathless! But as the case is not breathless and having pain most appropriate initial
action is analgesia].

Mock-june 2013. 102. A 54-year-old man comes with a sudden onset of palpitation and
breathlessness. His heart rate is 164 beats/minute. What is the single most appropriate
treatment in acute phase?

a. Adenosine
b. Metoprolol
c. Verapamil
d. Amiodarone
Ans. The key is C. Verapamil. [Though it can be thought as SVT but SVT has no acute or chronic
phase!! So the Dx is given in the question and it is acute phase of the arrhythmia; i.e. atrial
fibrillation (which has acute and chronic phase). Acute phase if hemodynamically stable (as in
presenting case: no feature of hemodynamical unstability is described) 1 st line treatment is
verapamil. Ref: OHCM].

Mock –june 2013. 103. A young adult presents to the A&E after a motorcycle crush. The patient
has bruises around his left orbital area. His GCS is 13 on examination and a strong alcoholic
breath is noticed. Shortly afterwards, His GCS drops to 7. What is the single most important
initial assessment?

a. MRI brain
b. CT brain
c. Chest X-ray
d. CT angiogram brain

Ans. The key is B. CT brain. [Young adult with considerable head injury (like RTA, fall from hight
etc.) with drop of GCS indicates acute extradural hematoma for which CT brain is the
investigation of choice].

Mock-june 2013. 104. A 27-year-old woman at 34 weeks gestation in her first pregnancy
attends ANC. Her blood results showed Hb. 10.6, MCV: 95, MCHC: 350. What should you do for
her?

a. Folate
b. Dextran
c. Ferrous sulphate
d. None

Ans. The key is C. Ferrous sulphate. This is a wrong key! Correct key is D. None. [According to
NICE, cut offs for iron supplements:
at booking (8-10 weeks)- if less than 11
at 28 weeks and further- if less than 10.5
if less than these values=> give iron].

Mock-june 2013. 105. A 60-year-old man is on treatment for ischemic heart disease and
hypertension and hyperlipidemia. During the night, he complains of wheezing and shortness of
breath. Which of the following medication is responsible for his symptoms?

a. Amlodipine
b. Atenolol
c. Ramipril
d. Simvastatin
e. Bendroflumethiazide

Ans. The key is D. Simvastatin. [Shortness of breath is found among people who take
Simvastatin, especially for people who are female, 60+ old , have been taking the drug for < 1
month, also take medication Aspirin, and have High blood cholesterol]. [On the other hand
Atenolol though occasionally can cause bronchoconstriction in a few susceptible patient but as
cardioselective usually does not cause bronchoconstriction].

Mock-june 2013. 106. A 30-year-old man complains of pain in the loin with a blood pressure of
140/90 mmHg. He is found to have proteinuria and hematuria. What is the investigation would
you do to confirm the diagnosis?

a. Ultrasound of abdomen
b. ANCA
c. ANA
d. Urine microscopy and culture
e. Stool culture

Ans. The key is A. Ultrasound of abdomen. [Dx is glomerulonephritis. Ultrasound will detect
kidney size to determine whether kidney sizes are normal (acute glomerulonephritis) or
reduced (chronic glomerulonephritis)].

Mock-june 2013. 107. A 4-year-old girl is taken by her mother to the A & E with feeling of being
unwell, urinary urgency and 39.C temperature. What is the single next best investigation?

a. Catheter catch of urine


b. Clean catch of urine
c. Ultrasound
d. Intravenous urography (IVU)
e. Suprapubic catch of urine

Ans. The key is B. Clean catch of urine. [Unwell, urgency and fever are suggestive of UTI for
which next best investigation is clean catch of urine].

Mock-june 2013. 108. A 35-year-old lady comes with a 5-year-old child who has nystagmus,
tinnitus and deafness. The boy is found to have tumour in the cerebellopontine angle. On
examination, the boy is found to have numerous café-au-lait spots seen on the back. What is
the inheritance ratio of this condition?
a. 100%
b. 1:2
c. 1:4
d. 1:8
e. None

Ans. The key is B. 1:2. [Probable diagnosis is neurofibromatosis type 2. Mode of inheritance is
autosomal dominant. So the inheritance ratio is 1:2].

Mock-june 2013. 109. A 70 year old man with cerebral metastasis develops cerebral oedema
and is being given dexamethasone and antihypertensive drugs. His blood glucose rises to 17-18
mmol/l. What is the appropriate management?

a. Low dose insulin


b. Optimal dose of insulin
c. Glibenclamide
d. Stop dexamethasone

Ans. The key is A. Low dose insulin. [Dexamethason induced hyperglycemia is not usually
controlled tightly but with low dose insulin. Preferred method is night time basal dose (like with
glargine) at bed time and premeal short acting insulin prior to each meal like lispro].

Mock-june 2013. 110. A 53-year-old female presents with an acute painful hot knee joint. She is
known case of rheumatoid arthritis. On examination, the skin is red, tender and swollen. The
hamstring muscles are in spasm. Her temperature is 38.5.C and blood pressure is 120/80
mmHg. What is the single best next investigation?

a. Joint aspiration for cytology, culture and sensitivity


b. Joint aspiration for positively birefringent crystals
c. Joint aspiration for negatively birefringent crystals
d. Blood culture
e. Serum uric acid

Ans. The key is A. Joint aspiration for cytology, culture and sensitivity. [Acute painful hot knee
joint is suggestive of septic arthritis. Also underlying rheumatological disease like ra are more
prone to septic arthritis. So the single best next investigation is joint aspiration for cytology,
culture and sensitivity].
***Mock-june 2013. 111. A 75-year-old man complaining of dripping incontinence and has an
ammonia smell. On examination, He has a furred tongue and a painless abdominal mass with
extension to the umbilical level. Urine analysis shows proteins ++ and nitrates + with no growth.
What is the most appropriate management for this patient?
a. Suprapubic catheterization
b. Trimethoprim
c. Intraurethral catheterization
d. Laparotomy
e. Condom catheter

Ans. C. Intraurethral catheterization. [Patient’s urine has no growth! So no need for antibiotics.
As there is overflow incontinence and bladder remains full, condom catheter is not suitable but
intraurethral catheter].

Mock-june 2013. 112. What is the most appropriate antibiotic to treat uncomplicated
chlamydia infection in a 21-year-old female who is not pregnant?

a. Erythromycin
b. Ciprofloxacin
c. Metronidazole
d. Cefixime
e. Doxycycline

Ans. The key is E. Doxycycline.

Mock-june 2013. 113. A child complains of right iliac fossa pain and diarrhea. On colonoscopy
granular transmural ulcers are seen near the iliocecal junction.

a. Sulfasalazine
b. Paracetamol
c. Ibuprofen
d. Metronidazole
e. Steroid

Ans. The key is A. Sulfasalazine. This is wrong key! Correct key is E. Steroid. [This is a case of
Crohn’s disease. In Crohn’s disease Steroids are 1st line. Ref: NHS.uk]

Mock-june 2013. 114. A 42-year-old man has been tired and sleepy for the last few weeks in the
morning. His work has started being affected as he feels sleepy in the meeting. His BMI is
36kg/m2. What is the single most likely diagnosis?

a. Idiopathic narcolepsy
b. Narcolepsy
c. Chest hyperventilation syndrome
d. Obstructive sleep apnoea syndrome
e. REM related sleep disorder

Ans. The key is D. Obstructive sleep apnoea syndrome. [Daytime somnolence and high BMI].

Mock-june 2013. 115. A 47-year-old man with a history of ischaemic heart disease complains of
chest pain with shortness of breath on exertion over the past few days. ECG is normal and
echocardiography shows decreased ejection fraction and decreased septal wall thickening.
What is the single most likely diagnosis?

a. Dilated cardiomyopathy
b. Constrictive pericarditis
c. Amyloidosis
d. Subacute endocarditis

Ans. The key is A. Dilated cardiomyopathy. [Diagnosis is ischemic dilated cardiomyopathy].

Mock-june 2013. 116. A 76 year old woman on paroxetine has increasingly become confused
and weak. What is the single most likely reason for her confusion?

a. Hypercalcemia
b. Hypernatremia
c. Hypocalcemia
d. Hypoglycemia
e. Hyponatremia

Ans. The key is E. Hyponatremia. [A well known complication of paroxetine therapy is SIADH
causing hyponatremia. Therefore we should be alert to hyponatremia in patients on paroxetine
by carrying out periodic monitoring of serum electrolytes, especially in elderly patients].

Mock-june 2013. 117. A 46-year-old man is being investigated for indigestion, jejunal biopsy
shows deposition of macrophages containing PAS positive granules. What is the most likely
diagnosis?
a. Bacterial overgrowth
b. Celiac disease
c. Tropical sprue
d. Whipple’s disease
e. Small bowel lymphoma

Ans. The key is D. Whipple’s disease. [Macrophages containing PAS positive granules on jejunal
biopsy is diagnostic of Whipple’s disease].
Mock-june 2013. 118. A 30-year-old woman has IUCD inserted 8-9 months ago. Now on routine
follow-up the thread is missing. Uterine ultrasound shows no IUCD in the uterus. What is the
best management?

a. Laparoscopy
b. Pelvic CT scan
c. Laparotomy
d. Pelvic X-ray
e. MRI

Ans. The Key is D. Pelvic X-ray. [Management of Lost Thread: Ultrasound should be arranged to
locate the device. If ultrasound does not locate the device and there is no definite history of
expulsion then abdominal X-ray should be performed to look for an extrauterine device.
Expulsion should not otherwise be assumed. Hysteroscopy can be helpful if ultrasound is
equivocal. Surgical retrieval of an extrauterine device is advised].

Mock-june 2013. 119. A man with dementia has an ulcerative lesion on his forehead. He wants
it to be removed so that “it can improve his memory.” The wife says he is not fit to give
consent. What will you do?

a. Get the GP to sign the consent


b. Get the wife to sign the consent
c. Get the patient to sign the consent
d. Refer to psychiatrist to assess the mental capacity to give consent

Ans. The key is D. Refer to psychiatrist to assess the mental capacity to give consent.

Mock-june 2013. 120. A 6-year-old-boy is brought to the A&E with pain in his left arm following
a fall from a tree on his outstretched hand. There is a swelling around his left elbow and a
reduced radial pulse. What is the most likely diagnosis?

a. Angulated fracture
b. Compound fracture
c. Epiphyseal fracture
d. No fracture
e. Spiral fracture

Ans. The key is ?? Given keys have not the correct answer!! The correct answer is
Supracondylar fracture.

***Mock-june 2013. 121. A young woman fainted in the street and was brought the A&E. She
weighs 33 kg. She is alert and well in the A&E and states she is on a diet and admits to not
eating or drinking anything for 48 hours. She is annoyed that she has been brought to the
hospital and adamantly states that she wants to leave. What is the most appropriate next step?

a. Detain her under the Mental Health Act


b. Refer to the dietician
c. Allow her to be discharged
d. Refer her to the GP

Ans. The key is A. Detain her under the Mental Health Act.

Mock-june 2013. 122. A woman with mild ulcerative colitis is not on any treatment. She
develops diarrhea and mucous in the last three months. All other examinations are normal. Her
Hb is 9.6 gm/dl. What is the single best treatment?

a. Magnesium sulphate
b. Ferrous sulphate
c. Meselazine
d. Endoscopy
e. Colonoscopy

Ans. The key is C. Meselazine. [Mesalazine - 5-aminosalicylic acid (5-ASA) - is now the treatment
of choice for induction and maintenance of remission of mild-to-moderate ulcerative colitis.
Oral mesalazine is less effective than oral corticosteroids and so should be used as sole
treatment only in mild attacks. Topical mesalazine is probably slightly more effective than
topical corticosteroids. Ref: patient.info].

Mock-june 2013. 123. A smoker with several ulcers in the mouth and lower surface of the
tongue and also with white striae on the mouth.

a. Aphthous ulcer
b. Lichen planus
c. Kaposis sarcoma
d. Cancer of the mouth

Ans. The key is D. Cancer of the mouth. [Smoking is a risk factor for cancer of mouth and oral
ulcers are common presenting feature of mouth cancer].

Mock-june 2013. 124. A middle aged man complains of a node which has been growing for
several months. Now it is firm and centrally depressed. What is the most likely diagnosis?

a. Basal cell carcinoma


b. Squamous cell carcinoma
c. Pyoderma gangrenosum
d. Melanoma
e. Necrobiosis lipoidica

Ans. The key is A. Basal cell carcinoma.

Mock-june 2013. 125. A 76-year-old man presents with loss of sensation on the middle part of
the chin and lower lip. Which nerve is involved?

a. Lingual
b. Inferior alveolar
c. Auricular
d. Submental
e. Buccal nerve.

Ans. The key is D. Submental.

Mock-june 2013. 126. A 37-year-old man had hemicolectomy. Immediately after operation, he
becomes aggressive not allowing doctors to give him analgesia. 2 days later he thanked the
doctor.

a. Delirium
b. Acute stress reaction
c. Depression
d. Adjustment disorder
e. Alcohol withdrawal

Ans. The key is B. Acute stress reaction. [Acute stress reaction (also called acute stress disorder,
psychological shock, mental shock, or simply shock) is a psychological condition arising in
response to a terrifying or traumatic event (here surgery), or witnessing a traumatic event that
induces a strong emotional response within the individual].

Mock-june 2013. 127. A 60-year-old lady has had breast surgery done for breast cancer. Which
one of the following will increase the chance of recurrence?

a. Sensitivity to estrogen receptors


b. Lymph node involvement
c. Presence of BRA1 and 2 gene
d. Family history
e. Age

Ans. The key is B. Lymph node involvement. [Risk factors. For breast cancer survivors, factors
that increase the risk of a recurrence include: Lymph node involvement. Finding cancer in
nearby lymph nodes at the time of your original diagnosis increases your risk of the cancer
coming back].
Mock-june 2013. 128. A 24-year-old schizophrenic man has recently been depressed. He is on
treatment for schizophrenia and depression. Which one of the following increases his risk for
suicide?

a. Alcohol
b. Fluoxetine
c. Haloperidol
d. Carbamazepine
e. Olanzepine

Ans. The key is A. Alcohol. [Alcohol and SSRIs if consumed together has synergistic effect to
raise suicidal risk. So alcohol is answer here as the patient is already getting fluoxetine].

Mock-june 2013. 129. A man has been diagnosed with indirect inguinal hernia. Which is the
single feature of the hernia sac would confirm the diagnosis?

a. Comes through the femoral ring


b. Passes through the deep inguinal ring
c. Lies below the lateral pubic tubercle
d. Does not pass through the deep inguinal ring

Ans. The key is B. Passes through the deep inguinal ring.

Mock-june 2013. 130. A 22-year-old man presented with chronic diarrhea for months.
Sygmoidoscopy shows ulcerative strictures. What will be the skin manifestation?

a. Pyoderma gangrenosum
b. Erythema multiforme
c. Necrobiosis lipoidica
d. Erythema chronicum migrans

Ans. The key is A. Pyoderma gangrenosum. [Pyoderma gangrenosum (PG) is the second most
common EIM that affects the skin of IBD patients. It seems to affect ulcerative colitis and
Crohn's disease (IBD) patients in roughly equal numbers. Fortunately, according to one recent
estimate, it affects less than one percent of IBD patients. Most common skin manifestation is
erythema nodosum].

Mock-june 2013. 131. Each year 950000 peaple develop MI in a population of 250000 million.
Out of these, 215000 die and 112500 in the first hour. What is the incidence of MI?

a. 112500 in 215000 million


b. 112500 in 250000 million
c. 215000 in 950000 million
d. 215000 in 250000 million
e. 950000 in 250000 million

Ans. The key is E. 950000 in 250000 million.

Mock-june 2013. 132. A 70-year-old man with metastatic cancer is on steroid therapy. His U&E
shows sodium 131, Potassium 5.5, and urea 12. What is the most likely underlying condition?

a. Adrenal insufficiency
b. SIADH
c. Hyponatremia
d. Hypernatremia
e. Hypokalemia

Ans. The key is A. Adrenal insufficiency. [Exogenous steroid causes suppression of


hypothalamic-pituitary-adrenal axis and any precipitating cause like surgery, infection etc. can
lead to adrenal insufficiency. In adrenal insufficiency there is hyponatremia and hyperkalemia].

Mock-june 2013. 133. An 8-year-old swallowed 12 tablets of paracetamol 4 hours ago. Serum
paracetamol levels when tested were at critical level. What is the single most likely
management?

a. Activated charcoal
b. IV N-acetyl systine
c. Gastric laage
d. Observation only

Ans. The key is B. IV N-acetyl systine.

Mock-june 2013. 134. A 79-year-old active man comes to you with complaints of swelling and
pain on dorsiflexion of the leg. On examination, there is oedema and a 3 cm increase in
diameter of the right cuff on comparison with the left cuff. What is the likely cause?

a. Popliteal artery aneurysm


b. Deep venous thrombosis
c. Popliteal cyst
d. Lymph oedema
e. DMD

Ans. The key is B. Deep venous thrombosis. Swelling of leg, pain on dorsiflexion of foot and
increase in diameter of cuff are suggestive of DVT].

Mock-june 2013. 135. A pregnant patient who’s Rh negative and has not has been previously
sensitized delivers her first baby without complications. What would be the latest time to
administer anti-sensitization?
a. 6 hours post partum
b. 24 hours post partum
c. 48 hours post partum
d. 72 hours post partum
e. 5 days post partum

Ans. The key is D. 72 hours post partum.

Mock-june 2013. 136. A 64-year-old man complains of increasing shortness of breath and cough
for the past 18 months. He cough up a tablespoon of mucopurulent sputum with occasional
specks of blood. What is the most likely underlying cause?

a. Acute bronchitis
b. Bronchiectasis
c. Chronic bronchitis
d. Lung cancer
e. Pneumonia

Ans. The key is B. Bronchiectasis. This is a wrong key. Correct key is C. Chronic bronchitis. [In
bronchiectasis daily sputum production is cupful not only a tablespoon!! Though 18 month does
not fulfill the time requirement to make the diagnosis of chronic bronchitis yet but it is the most
likely diagnosis as given info].

Mock-june 2013. 137. A 28-year-old man believes that each time the traffic lights change, it
means that his mother needs to see him urgently and he needs to return home. He has only
slept for 2 hours in the last two days and eats very little. He believes that his thoughts are being
taken out of his head and are being made available for the people around him. What is the
single most likely diagnosis?

a. Depression
b. GAD
c. Mania
d. Obsessive compulsive disorder
e. Schizophrenia

Ans. E. Schizophrenia. [Delusion, poor sleep, little eating and thought broadcasting are features
of schizophrenia].

Mock-june 2013. 138. A 7-year-old girl comes with dysuria and fever of 37.8 C. What is the most
appropriate investigation?

a. Clean catch of urine


b. Full blood count
c. Renal ultrasound
d. Suprapubic aspirate

Ans. The key is A. Clean catch of urine. [dysuria, fever points towards UTI for which clean catch
of urine is the investigation of choice].

Mock-june 2013. 139. A 28-year-old woman who is breast feeding has pain in her left breast.
The outer upper quadrant of the breast is red, swollen and tender.

a. Staphylococcus aureus
b. Staphylococcus epidermidis
c. Streptococcus feacalis
d. Streptococcus pyogenes
e. Streptococcus viridens

Ans. The key is A. Staphylococcus aureus.

Mock-june 2013. 140. A 70-year-old man present with sudden onset of severe pain in the lower
part of his abdomen. He is unable to pass water and distressed. On examination, the abdomen
is distended, tender and dull on percussion. What is the single most appropriate initial
management?

a. Arranging abdominal X-ray


b. Giving morphine
c. Inserting urinary catheter
d. Sending blood for amylase
e. Starting IV fluid

Ans. The key is C. Inserting urinary catheter. [A case of acute retention. So urinary catheter
should be inserted].

***Mock-june 2013. 141. A 28-year-old woman suddenly develops a red rash all over the body
following a wasp sting. Her pulse is 100 beats/minute, BP 120/70 mmHg, RR 24c/m. What is the
single most important clinical feature indicative of adrenaline?

a. IM adrenaline 0.5 ml in 100


b. IM adrenaline 0.5 ml in 1000
c. IM adrenaline 0.5 ml in 10,000
d. IM adrenaline 0.5 ml in 100,000
e. IM adrenaline 0.5 ml in 1,000,000

Ans. The key is B. IM adrenaline 0.5 ml in 1000.


Mock-june 2013. 142. A 72-year-old man has had cough and fever for the past 1-week and now
has suddenly become short of breath and coughed up a cup full of fresh blood. His temperature
is 38.6 C, HR 126 beats/min, saturations 96% on 10L, BP 150/85 mmHg, RR 43c/min. IV
cannulation has been done. Which is the most appropriate next step?

a. Antibiotics
b. Arterial blood gas
c. Blood culture
d. Chest X-ray
e. ECG

Ans. The key is A. Antibiotics. [The patient has developed pneumonia. Pneumonia can cause
massive hemoptysis and the treatment of pneumonia is antibiotics].

Mock-june 2013. 143. A 20-year-old lost control of her car and hit a tree while driving at a
speed of 60 mph. She has a large laceration on the parietal scalp and some bruises on the right
chest. Her GCS is 14/15 and she is speaking in sentences. She has been given high flow oxygen
via facemask. Which is the most immediate management?

a. Cervical spine immobilization


b. Chest tube insertion
c. CT brain scan
d. Endotracheal intubation
e. IV access

Ans. The key is A. Cervical spine immobilization. [Whiplash injury with trauma head. So injury to
cervical spine should give urgent importance and cervical spine immobilization should be done
early till cervical spinal injury is ruled out].

Mock-june 2013. 144. A 35-year-old man skidded on a wet road while riding his motorbike at a
speed of 70 mph. He has a large hematoma on the temporal scalp, some bruises on the chest
wall and abdomen and a deformed thigh. His GCS is 11/15. He has been given high flow oxygen
via facemask. Which is the single most immediate radiological investigation required during the
initial resuscitation phage?

a. CT abdomen
b. CT brain
c. X-ray of the chest
d. X-ray of the abdomen
e. X-ray of the femur

Ans. The key is B. CT brain. [35 yrs old man with significant head trauma from RTA with low GCS
is highly suggestive of extradural hematoma for which CT brain should be done].
Mock-june 2013. 145. A 20-year-old man’s house caught fire. He has blisters and peeling of the
skin. He is coughing black sputum. What is the single most appropriate immediate
management?

a. Arrange chest X-ray


b. Applying dressing on the face
c. IM analgesia
d. Transfer to burn unit
e. Establish a definitive airway.

Ans. The key is E. Establish a definitive airway. [Coughing black sputum (soot in RTI) indicate
inhalation injury and if a burn patient has inhalation injury immediate measurement to be
taken is establish a definitive airway].

***Mock-june 2013. 146. A 50-year-old man present with central crushing chest pain radiating
to the left arm. ECG is shown below. What is the next step in management?

a. Warfarine
b. Low molecular weight heparin
c. Percutaneous coronary intervention
d. Aspirin
e. Clopidogrel
Ans. The key is C. Percutaneous coronary intervention. This is wrong key! Correct key is D.
Aspirin. [In IHD we give aspirin 300mg stat and then proceed to other procedures].

Mock-june 2013. 147. A 60-year-old woman present with post coital bleeding for the past 1
week. What is the most appropriate investigation?

a. Cervical smear
b. Cervical biopsy
c. Colposcopy
d. Endometrial sampling
e. Transvaginal Ultrasound

Ans. The key is C. Colposcopy. [Please note Red Flag Sign for Cancer (either cervical or
endometrial) is AGE>35 yrs and persistence of post coital bleeding for more than 4 WEEKS!! So
the important clincher here is duration and presenting case still is not supposed to be
diagnosed as cancer!! Hence infection is most likely cause or atrophic vaginitis and from the
given options C. Colposcopy is the likely correct option].

Mock-june 2013. 148. A 2-year-old boy has a temperature of 39.C and purulent otorrhoea. His
pinna is laterally and inferiorly displaced. Which is single most appropriate management?

a. Admit for analgesia


b. Admit for intravenous antibiotics
c. Emergency grommet insertion
d. Antibiotic ear drops
e. Oral antibiotics

Ans. The key is B. Admit for intravenous antibiotics. [age and severity of the disease needs iv
antibiotics].

Mock-june 2013. 149. A 32-year-old woman is having intermittent episodes of vertigo. Each
episodes lasts up to 12 hours and is associated with tinnitus and hearing loss. Which is the
single medication that may help in reducing the frequency of the vertigo episode?

a. Amitriptyline
b. Paroxetine
c. Cyclizine
d. Betahistine
e. Prochlorperazine

Ans. The key is D. Betahistine. [Case described is Menier’s disease and to reduce the frequency
of vertigo episodes (Prophylaxis) betahistine is given].
Mock-june 2013. 150. A 72-year-old woman with rheumatoid arthritis has horseness of voice.
There is left vocal cord palsy. Which is the joint likely to be involved?

a. Atlanto-occipital
b. Cricoaretinoid
c. Cricothyroid
d. Costrochondral
e. Sternoclavicular

Ans. The key is B. Cricoaretinoid.

Mock-june 2013. 151. A 68-year-old woman has otalgia and dysphagia. She has angular cheilitis
and pale conjunctiva. Her oropharyngeal examination is normal. In which single anatomical site
is this patient likely to have a tumour?

a. Lower oesophagus
b. Nasopharyngeal
c. Post cricoids
d. Thyroids
e. Tonsil

Ans. The key is C. Post cricoid. [Most patients with post cricoid tumour report a globus and
dysphagia more to solid than liquids and gradually develops in odynophagia. Progression of
tumour causes otalgia which is an ominous sign. Conjunctival pallor is due to tumour related
anemia].

Mock-june 2013. 152. A 4-year-old boy with Down’s syndrome has proven middle ear effusion
bilaterally that has been present for 6 months. Which is the single most appropriate
management?

a. Adenoidectomy
b. Bilateral mastoidectomy
c. Bilateral grommet ventilation
d. Cochlear implantation
e. Watch and wait for three months

Ans. The key is C. Bilateral grommet ventilation. [Previously grommet insertion was done if fluid
persisted beyond three months. Now this concept is changing and degree of hearing loss (a loss
of 40 dB or greater is felt to be an absolute indication and a loss in the range of 21-40 dB is a
relative indication) is considered for grommet placement].

Mock-june 2013. 153. A 20-year-old man has a chronic facial palsy and rinorrhoea. He has
multiple courses of antibiotics with little improvement in his symptoms. A CT scan shows a
round opacity with mixed density within the right maxillary sinus. What is the most likely
diagnosis?

a. Allergic
b. Angiofibroma
c. Antrochoanal
d. Fungal ball
e. Foreign body within the sinus

Ans. The key is D. Fungal ball. [Failure of multiple courses of antibiotics and CT findings are
suggestive of fungal ball].

Mock-june 2013. 154. A 78-year-old man has sudden onset of horseness of voice. He has a
weak cough and he coughs when eating. He has been a smoker since 18 years ago. He has right
sided vocal cord palsy. Which is the single most likely anatomical site of his primary tumour?

a. Bronchus
b. Larynx
c. Oesophagus
d. Oral cavity
e. Parotid

Ans. The key is B. Larynx. [Horseness of voice, weak cough due to vocal cord palsy and smoking
makes the likely diagnosis of laryngeal tumour].

Mock-june 2013. 155. A 22-year-old woman says she has taken about 40 tablets of paracetamol
3 hours ago. Her heart rate is 110 beats/minute, BP 110/80mmHg, and RR 22c/m. Which is the
single most appropriate initial management?

a. Give activated charcoal


b. Give N-acetylcystine
c. Induce vomiting
d. Perform gastric lavage
e. Wait for the 4 hour paracetamol level

Ans. The key is E. Wait for the 4 hour paracetamol level. [NICE guideline].

***Mock-june 2013. 156. A 30-year-old woman says she has taken about 34 tablets of
paracetamol about 12 hours ago. She is now unwell. Her heart rate is 120 beats/min, BP
112/78mmHg and RR 20c/m., which is the single most important management?

a. Discuss with the liver transplant unit


b. Give N-acetylcysteine immediately
c. Give activated charcoal immediately
d. Refer for psychiatric assessment
e. Wait for the paracetamol level

Ans. The key is B. Give N-acetylcysteine immediately. [Dangerous dose is 24 tablets and if
during 8-15 hours from ingestion start treatment with N-acetylcysteine immediately].

Mock-june 2013. 157. A 20-year-old woman says she has taken 40 tablets of paracetamol over
the past 3 days. She is not unwell and has vomited a few times. Her heart (??probably RR) rate
is 22 beats/min. The blood tests are as follows:

Sodium 146mmol/L, Potassium 3.5mmol/L, Urea 5.6mmol/L, Creatinine 115micromol/L,


Albumin 32g/L, Bilirubin 52micromol/L, ALT 2560IU/L, Alkaline phosphatase 230IU/L, Gamma
transpeptidase 230IU/L, pH 7.3, INR 4, Lactate 4.2mmol/L, Paracetamol 190mg/L. Which is the
single most likely chemical compound responsible for this findings?

a. NAPQI
b. N para-acetyl aminophenol
c. N para-acetyl aminophenol glucoronate
d. Para-acetyl aminophenol sulphate
e. Para-amino phenol

Ans. The key is A. NAPQI. [NAPQI (N-acetyl-p-benzoquinone imine) is a toxic byproduct


produced during the xenobiotic metabolism of paracetamol (acetaminophen)].

Mock-june 2013. 158. A 36-year-old woman has taken about 28 tablets of paracetamol 6 hours
ago. She has been drinking a bottle of vodka and a couple pints of lager everyday for the last
few years. She feels nauseous and has vomited a few times.
Sodium 144mmol/L, Potassium 3.8mmol/L, Urea 5.6mmol/L, Creatinine 86micromol/L, Albumin
30g/L, Bilirubin 15micromol/L, ALT 30IU/L, AST 32IU/L, Alkaline phosphatase 230IU/L. Gamma
glutamile transpeptidase 1261IU/L, Paracetamol 80 mg/L
Which is the single most appropriate immediate management?

a. Discussing with liver transplant unit


b. Give acetylcysteine
c. Give fresh frogen plasma
d. Give vitamin K injection
e. Repeat the blood tests

Ans. The key is B. Give acetylcisteine. [Management at 4-8 hours from ingestion: Take drug
levels immediately and use graph. As the paracetamol level is high here (80mg/L) give
acetylcystine].

Mock-june 2013. 159. A 40-year-old man with chronic atrial fibrillation presents with
palpitations. What is the most appropriate treatment?
a. Metoprolol
b. Digoxin
c. Verapamil
d. Atenolol
e. Amlodipine

Ans. The key is A. Metoprolol.

Mock-june 2013. 160. A 67-year-old man presents with shortness of breath. On examination, he
has a harsh pansystolic murmur at the apex which radiates to the axilla. What is the cause.

a. Ventricular septal defect


b. Mitral stenosis
c. Tricuspid regurgitation
d. Mitral stenosis (?)
e. Aortic regurgitation

Ans. The key is C. Tricuspid regurgitation.

***Mock-june 2013. 161. A healthy baby boy is born at term to a woman who was unwell with
confirmed acute hepatitis B during pregnancy. The mother is very concerned that she may have
infected the baby with hepatitis B. What is the single preventive measure should be given to
the baby?

a. Full course of hepatitis B vaccine


b. Hepatitis B immunoglobulin alone
c. Hepatitis B status testing
d. Hepatitis B vaccine and hepatitis B immunoglobulin
e. Hepatitis vaccine as single dose

Ans. The key is D. Hepatitis B vaccine and hepatitis B immunoglobulin.

Mock-june 2013. 162. An 18 months girl who has had a single urinary tract infection is seen in
the outpatient department. She has fever and vomiting but these improved with a course of
trimethoprim. Urine culture became negative. Subsequently a micturating systogram showed
bilateral vesico-ureteric reflux. What is the single most appropriate management?

a. No treatment
b. Oxybutinin (anti-cholinergic)
c. Potassium citrate (urinary alkanisation)
d. Prophylactic antibiotic
e. Ureteric surgery
Ans. The key is E. Ureteric surgery. This is a wrong key! Correct key should be D. Prophylactic
antibiotic. [Majority of VUR get cured spontaneously with prophylactic antibiotics within 5 years
of age and only a minority needs surgery].

Mock-june 2013. 163. A 6-year-old boy presents with a fever and is unable to swallow his saliva.
He has a temperature of 39.C, pulse rate of 130 beats/minute and RR of 40c/m. What is the
single most appropriate initial management?

a. Emergency help from anaesthetist or ENT surgeon


b. Establish intravenous (IV) access and give IV penicillin
c. Examine the throat using a tongue depressor
d. Lay the patient flat and give high flow oxygen
e. Obtain lateral soft tissue neck x-ray

Ans. The key is A. Emergency help from anaesthetist or ENT surgeon. [Probable dx is acute
epiglottitis. Initial management is to establish intubation to save the child from closure of
airway which frequently occurs in epiglottitis].

Mock-june 2013. 164. A 78-year-old man presents to the A&E department with an upper
gastrointestinal hemorrhage. According to his wife, he is taking tablets for osteoporosis to build
his bones and volarol for pain relief. The following are causes of gastric or lower oesophageal
ulceration except:

a. Crohn’s disease
b. Celecoxib
c. Alendrnate
d. Misoprostol
e. Chronic renal failure

Ans. The key is D. Misoprostol. [Misoprostol helps prevent stomach ulcer].

Mock-june 2013. 165. A town has population of 500000. In a 5 year period, 100 people present
with cancer Y. During the same period, 1250 cases of cancer Y were registered by death
certification. And also it was found that in the same city 500 people were found to have cancer
Y from the community hospital. The 1-year survival rate is 0%. What is the annual prevalence of
cancer Y per million in this population?

a. 200
b. 400
c. 500
d. 700
e. 1250

Ans. The key is D. 700. [Probably the question is not correct as the result comes as 740].
***Mock-june 2013. 166. A 40-year-old patient with malabsorption had biopsy done. On biopsy
she was found to large amount of biopsy of lymphocytes. What is the single most likely
diagnosis?

a. Whipples disease = crohn’s disease


b. Ulcerative disease
c. Celiac disease = increased lymphocytes
d. Helicobacter pylori
e. Gastric cancer

Ans. C. Celiac disease = increased lymphocytes.

Mock-june 2013. 167. A 34-year has been drinking for 4 days, now he comes saying that he can
feel insect walking on the skin and nasal redness. What is the single most likely drug abuse?

a. Alcohol
b. Cocaine
c. LSD
d. Heroin
e. Cannabis

Ans. The key is A. Alcohol.

Mock-june 2013. 168. A 40-year-old man presents with abdominal distension 4 days after an
open cholecystectomy. On examination there were no bowel sound. What is the diagnosis?

a. Intestinal obstruction
b. Paralytic ileus
c. Internal bleeding
d. Anastomotic leak
e. Abdominal aneurysm

Ans. The key is B. Paralytic ileus. [Abdominal distension after surgery with absence of bowel
sound suggests paralytic ileus].

Mock-june 2013. 169. A 57-year-old man with known diabetes has cholesterol 5.7. Which of the
following is the correct combination of medication for his management?

a. Angiotensin converting enzymes and statin


b. Angiotensin converting enzyme and metformin
c. Angiotensin converting enzyme and glibenclamide
d. Angiotensin converting enzyme and gliclazide
e. Glibenclamide and statin
Ans. The key is E. Glibenclamide and statin. [This is an incomplete question. Whether we should
give sulfonylurea or glibenclamide depends upon BMI. If BMI is <25 we give sulfonylurea and if
>25 we give glibenclamide].

Mock-june 2013. 170. A 45-year-old man presents to accident and emergency department with
headache. He is then found to have high blood pressure of 210/110. His blood results are as
follows: Potassium is 2.7, Urea 4.5, creatinine 124, Sodium 136. What is the single most
appropriate investigation?

a. Cortisol
b. Aldosterone
c. Adrenaline
d. Renin
e. Growth hormone

Ans. The key is B. Aldosterone. [Hypercortisolism can present with similar findings but features
like moon face, buffalo hump, etc. makes the differentiating features].

Mock-june 2013. 171. An 18-year-old man presents with sudden onset of testicular pain. He has
history of multiple sexual partner. Temperature 37.4, blood pressure 120/80. There is no
urethral discharge. What is the single most likely diagnosis?

a. Testicular torsion
b. Epididymo-orchitis
c. Testicular tumour
d. Epididymal cysts
e. Hydrocele

Ans. The key is A. Testicular torsion. [Sudden onset, normal temperature, and absence of
urethral discharge suggests testicular torsion].

Mock-june 2013. 172. A 45-year-old man presents with ascites and epigastric pain. He had
peritoneal lavage done which shows protein less than 25. What is the single most likely cause of
ascites?

a. Tuberculosis
b. Cirrhosis
c. Bud chiari syndrome
d. Alcoholic liver disease
e. Carcinomatosis

Ans. The key is B. Cirrhosis.


Mock-june 2013. 173. A 65-year-old man presents with ascites. After peritoneal lavage he is
found to have low glucose and red cells. What is the single most likely cause?
a. Gastric cancer
b. Tuberculosis
c. Bud chiari syndrome
d. Liver cirrhosis
e. Heart failure

Ans. The key is A. Gastric cancer. [Both gastric cancer and tuberculosis has the similar picture.
TB can occur in any age but cancer usually occurs in elderly].

Mock-june 2013. 174. A 30-year-old was put on a theater table having an operation with his
hand extended. After the operation he is unable to extend his wrist and he also has loss of
sensation on the first dorsal area between the thumb and index finger. Which nerve is most
likely to be affected?

a. Radial nerve
b. Median nerve
c. Axillary nerve
d. Musculocutaneous

Ans. The key is A. Radial nerve. [Symptoms (and signs) of radial neuropathy vary depending on
the severity of the trauma; however, common symptoms may include wrist drop, numbness
(back of the hand and wrist), and inability to voluntarily straighten the fingers. Loss of wrist
extension is due to loss of the ability to move of the posterior compartment of forearm
muscles].

Mock-june 2013. 175. A 51-year-old man was sitting for few hours with his legs crossed. Later
he could not dorsiflex or invert is foot. He also has got loss of sensation on the dorsum of his
foot. Which nerve is most likely to be damaged?

a. Peroneal nerve
b. Sciatic nerve
c. Sural nerve
d. Femoral nerve
e. Tibial nerve

Ans. The key is A. Peroneal nerve. [Symptoms of peroneal nerve injury (foot drop) may include:

 Inability to point toes toward the body (dorsi flexion)


 Pain
 Weakness
 Numbness (on the shin or top of the foot)
 Loss of function of foot
 High-stepping walk (called steppage gait or footdrop gait)].

Mock-june 2013. 176. A mother brings her 2 days old baby worried she could be deaf. The
mother is deaf herself and there is family history of deafness as well. What is the single most
appropriate investigation?

a. Brainstem evoked response


b. Distraction
c. Tuning fork tests
d. Startle reflex
e. Audiogram

Ans. The key is A. Brainstem evoked response. [Birth to 6 months, Brainstem evoked response].

Mock-june 2013. 177. A 45-year-old lady presents with back pain to accident and emergency
department. She had a lumbosacral X-ray done which is normal. On examination she is found to
have loss of sensation of the lower legs on pin prick. What is the most appropriate investigation?

a. Bone nuclear scan


b. MRI scan
c. Skeletal survey
d. CT scan spine
e. No investigation

Ans. The key is B. MRI scan. [Probable case of disc herniation for which MRI is the
investigation of choice].

Mock-june 2013. 178. A 50-year-old lady with history of breast cancer treated with
chemotherapy and surgery. Now she has come back with pelvic pain. What is the most
appropriate management?

a. DEXA
b. Radiotherapy
c. Additional chemotherapy
d. Bisphosphonates
e. Mamography

Ans. The key is B. Radiotherapy. [External beam radiotherapy and systemic endocrine and
cytotoxic treatments are the mainstay of treatment in advanced cancers. However, it is now clear
that the bisphosphonates provide an additional treatment strategy, which reduces both the
symptoms and complications of bone involvement].
Mock-june 2013. 179. A 2-year-old child was found to have meningitis. The child recovered
well. He was not resuscitated. What would be the expected outcome of the child?

a. Goes back to normal status like before the illness


b. Hearing loss
c. Visual loss
d. Mental state retardation
e. Hemiperasis

Ans. The key is A. a. Goes back to normal status like before the illness.

Mock-june 2013. 180. A 35-year-old man presents with headache and rash. On examination the
rash does not blanch. What is the single most appropriate antibiotics?

a. Cefotaxime
b. Amoxicillin
c. Ciprofloxacin
d. Vancomycin
e. Clarithromycine

Ans. The key is A. Cefotaxime. [The likely diagnosis is meningococcal septicemia which is
treated with Cefotaxime (OHCM, 9th edition, page 833)].

***Mock-june 2013. 181. A 45-year-old man presents with fever, rash and headache. His
Glasgow coma scale is 10/15. What is the single most appropriate immediate management?

a. Intravenous antibiotics and CT scan brain


b. Lumber puncture and IV antibiotics
c. CT scan of the brain and intravenous fluids
d. Lumber puncture and antibiotics
e. Blood culture and lumber puncture

Ans. The key is A. Intravenous antibiotics and CT scan brain.

Mock-june 2013. 182. A 47-year-old man presents with fever, headache, neck stiffness and
photophobia. He is also vomiting. What is the single most diagnostic investigation?

a. Blood culture
b. CT scan brain
c. Blood culture
d. Lumber puncture
e. MRI brain

Ans. The key is D. Lumber puncture. [Though vomiting is a warning sign of possible raised
intracranial pressure but it is used as a distraction here. The question is not next or initial
investigation but the SINGLE MOST DIAGNOSTIC investigation which is lumber puncture. So
for this question the answer is D. Lumber puncture.].

Mock-june 2013. 183. A 40-year-old man presents to accident and emergency department with
confusion, ataxia and ophthalmoplegia. He lost his job recently and he is smelling of sweet
breath. Alcohol breath test is negative. What is the single most appropriate immediate
management?

a. Glucose
b. Intravenous fluid
c. Intravenous thiamine
d. Potassium chloride
e. Chlordiazepoxide

Ans. The key is C. Intravenous thiamine. [Classic triad of Wernicke’s encephalopathy. Treated
with intravenous thiamine].

Mock-june 2013. 184. A 40-year-old man lost his job 1 year ago. The police picked him from the
street. He has a sweet smell. Alcohol breath test was negative. What is the most likely
diagnosis?

a. Alohol intoxication
b. Alcohol withdrawal
c. Delirium tremens
d. Wernicke’s encephalopathy
e. Acute psychosis

Ans. The key is D. Wernicke’s encephalopathy. [As alcohol breath test is negative it is not
alcohol intoxicatin and police picked him and done alcohol breath test supports wernicke’s
encephalopathy (ataxia probably was noticed)].

Mock-june 2013. 185. An 18-year-old lady was found to have BMI of 22. She had lost 8kg of
weight in 6 months. She has low mood, she sad all the time. She then started to buy expensive
clothes and eating in expensive restaurant. What is the single most appropriate diagnosis?

a. Anorexia nervosa
b. Bulaemia nervosa
c. Bipolar affective disorder
d. Schizophrenia
e. Depression

Ans. The key is C. Bipolar affective disorder.


Mock-june 2013. 186. A 3-yr-old girl fell down while riding her bicycle. She then went pale and
then rigid. 2 minutes later she recovered fully but she was still pale. After 1 hour she well and
playing around. What is the most appropriate management?

a. No investigations
b. Electrocardiogram
c. Full blood count
d. Electroencephalogram
e. CT scan of the brain

Ans. The key is A. No investigations. [It is breath-holding spell which is transient and becomes
normal within few minutes. No investigation is needed].

Mock-june 2013. 187. A 19-year-old man does not talk to anyone. He has collected 2000 toy
cars. He is indifferent to his mum. What is the single most appropriate investigation?

a. Autistic spectrum disorder


b. Psychosis
c. Borderline personality disorder
d. Dissociative disorder
e. Schizotypal disorder

Ans. The key is A. Autistic spectrum disorder.

Mock-june 2013. 188. A 30-year-old lady every time she leaves the house experiences
palpitation and difficulty in breathing. She has to rush back into her house to allay her anxiety
with a drink. What is the single most likely diagnosis?

a. Social phobia
b. Agarophobia
c. Panic attack
d. Hysteria
e. Arachnophobia

Ans. The key is B. Agarophobia. [Agoraphobia is a fear of being in situations where escape might
be difficult or that help wouldn't be available if things go wrong.

Many people assume agoraphobia is simply a fear of open spaces, but it's actually a more
complex condition. Someone with agoraphobia may be scared of:

travelling on public transport

visiting a shopping centre


leaving home].

Mock-june 2013. 189. A 40-year-old lady was biten by a bee on her right hand 3 weeks ago. She
has past history of past medical history breast cancer for which she had radiotherapy and
surgery with axillary clearance. She presents to accident and emergency department with
swelling of the right hand. What is the single most likely diagnosis?

a. Anaphylaxis
b. Delayed allergic reaction
c. Cellulitis
d. Lymphoedema
e. Superior vena caval obstruction

Ans. The key is D. Lymphoedema. [axillary clearance may cause compromised lymphatic
drainage of upper limb leading to lymphoedema].

Mock-june 2013. 190. A 60-year-old lady who had anterior resection of his bowel 5 days ago
presents with swelling on her wound. There is 4cm swelling is around the colostomy which
tender and it fluctuating. She is not systematically unwell. What is the single most appropriate
action?

a. Laparotomy
b. Local anesthesia
c. Laparotomy with wound repair
d. Exploration of the wound
e. Abdominal X-ray

Ans. The key is D. Exploration of the wound.

Mock-june 2013. 191. A 25 year old lady presents to outpatient department requesting
contraception. She has irregular and painful periods. She smokes 20 cigarettes a day. Otherwise
she is fit and well. Which is the single most appropriate type of contraception?

a. Combined oral contraception


b. Progesterone only contraception
c. Trenexamic acid
d. Levenogestrel
e. Levanogestrel IUCD

Ans. A. Combined oral contraception [smoking >20 cigarettes at age >35yrs are contraindication
for COCP, In irregular period COCP is the drug of choice as this patient is <35 best choice for her
is A. Combined oral contraception].
Mock-june 2013. 192. A 16-year-old girl presents with heavy regular periods. What is the single
most appropriate treatment?

a. Combined oral contraception


b. Progesterone only pill
c. Trenexamic acid
d. Mefenamic acid
e. Levonogestrel intrauterine system

Ans. The key is C. Trenexamic acid. [As the patient did not ask for contraception or not
described as sexually active, trenexamic acid is best option for her].

Mock-june 2013. 193. A 30-year-old lady with past medical history of pulmonary embolism
presents to outpatient clinic requesting contraception. She has irregular and painful periods.
What is the single most appropriate contraception?

a. Intrauterine contraceptive device


b. Combined pill contraception
c. Progesterone only pill
d. Mefenamic acid
e. Trenexamic acid

Ans. The key is C. Progesterone only pill. [If irregular period COCP is the contraception of choice.
But as there is history of pulmonary embolism COCP should be avoided and next option is
progesterone only pill].

Mock-june 2013. 194. A 34-year-old man had a 4mm ureteric stone which he passed with urine.
This time he presents with 3cm stone in right kidney. What is the single most appropriate
treatment?

a. No treatment
b. Lithotripsy-shock wave
c. Laparotomy
d. Operative stone removal
e. Observe

Ans. The key is E. Observe. This is wrong key. Correct key is D. Operative stone removal. [kidney
stone is risk factor for infection and renal damage. So there is no scope for observation but
removal].
Mock-june 2013. 195. A 35-year-old man returned from Egypt where she was working in
outdoor. She was found to have a 5cm stone which was removed operative. She is now
planning to go back to Egypt. What advice would you give her?

a. Increase fluid intake


b. Better not to go to Egypt again
c. Reduce fluid intake
d. Regular hospital admission for intravenous fluids
e. Return to UK regularly

Ans. The key is A. Increase fluid intake.

Mock-june 2013. 196. A 28-year-old man with a known psychiatric disease thinks that he is the
professor of medicine and he says later he will become God. What is the single most likely
diagnosis?

a. Delusion of grandiose
b. Idea of reference
c. Identity disorder
d. Paranoid delusion
e. Delusion of reference

Ans. The key is A. Delusion of grandiose. [1. Delusion of grandiose- A delusion of grandiose is
the fixed, false belief that one possesses superior qualities such as genius, fame, omnipotence,
or wealth. 2. Idea of reference- The patient falsely believe that everything occurring around
them is related somehow to them when in fact, it isn't. 3. Identity disorder- Condition in which
two or more distinct identities, or personality states, are present in—and alternately take
control of—an individual. Paranoid delusion- Delusional disorder is a mental illness in which the
patient presents with delusions, but with no accompanying prominent hallucinations, thought
disorder, mood disorder, or significant flattening of affect. 5. Delusion of reference-
synonymous to Idea of reference described earlier].

Mock-june 2013. 197. A 30-year-old lady with past history of 1 episode of mania presents with
depression. She has been treated successfully this time but she needs prophylaxis for her
condition. What is the most appropriate medication/

a. Mood stabilizer
b. Benzodiazepines
c. Anti-depressant
d. Anti-psychotic
e. Anti-epileptic
Ans. The key is A. Mood stabilizer.

Mock-june 2013. 198. A 35-year-old diabetic patient was on long acting anti diabetic in the
morning and short acting in the evening. After lunch patient gets hypoglycemia. How you will
manage this patient?

a. Provide more food at lunch


b. Decrease long acting in the morning
c. Decrease short acting in the afternoon
d. Decrease both dose
e. Reassurance

Ans. The key is B. Decrease long acting in the morning.

Mock-june 2013. 199. A 70-year-old man with lung cancer presents with a cough. He is already
on codeine, steroid and salbutamol inhalers. What is the single most appropriate management?

a. Nebulized adrenaline
b. 0.9% saline nebulized
c. Oral morphine
d. Antibiotics
e. Benzodiazepines

Ans. The key is B. 0.9% saline nebulized. [Nebulized NS loosens the phlem and help to
expectorate it easily].

Mock-june2013. 200. A 65-year-old man presented with shortness of breath. He was a shipyard
worker. He was admitted treated with antibiotics and inhalers. What would be the reason to
inform the coroner?

a. Industrial disease
b. Inappropriate treatment
c. Because he died during admission
d. To satisfy the relatives
e. It should be done as a routine

Ans. The key is a. Industrial disease. [The coroner is an independent judicial officer. The
coroner's job is to establish why the person has died and the surrounding circumstances. The
coroner's involvement is unrelated to any claim for compensation. However, a coroner's verdict
of 'industrial disease', as the cause of death, can be used to support a claim].

You might also like